BMJ OnExamination Flashcards

1
Q

Regarding pulse oximetry, are the following statements true or false?

True / False
Hypothermia produces a left shift in the oxygen dissociation curve
Is unreliable when used on the same side as the blood pressure cuff
The relationship between the partial pressure of oxygen in arterial blood (PaO2) and percentage oxygen saturation (%SaO2) is linear
Have an accuracy of +/-2% in the range of oxygen saturations of 70% to 100%
Is unaffected by the carboxyhaemoglobin level

A

True
True
False
True
False

Pulse oximetry is a non-invasive method of assessing arterial oxygen saturation and heart rate.

Any cause of poor peripheral perfusion causes unreliable readings including external compression by a blood pressure cuff.

Pulse oximeters have an accuracy +/-2% in the range of oxygen saturations of 70% to 100%, however, they are less accurate below 70%. Below an oxygen saturation of 70% the readings are extrapolated.

Other causes of error include abnormal pigments such as:

Methaemoglobin
Carboxyhaemoglobin.
Carbon monoxide poisoning overestimates and methaemoglobin underestimates the level of oxygen saturation.

The literature is a little confused on whether hyperbilirubinaemia affects pulse oximetry. Current consensus is that it has little or no effect on the accuracy of readings

When the partial pressure of oxygen is plotted against percentage saturation, a sigmoidal curve is produced. The middle range of the curve is therefore particularly important, as small changes in partial pressure will cause large changes in saturation.

Certain factors may result in ‘shifts’ in the curve, such as:

Reduced pH
Increased temperature
Partial pressure of carbon dioxide and
2,3-diphosphoglycerates.
cause a right shift in the curve. This means that haemoglobin gives up oxygen more easily to the tissues.

How well did you know this?
1
Not at all
2
3
4
5
Perfectly
2
Q

Which one of the following is a primary perioperative concern for patients taking chlorpromazine?

Reduction in anaesthetic agent requirements
Prolongation of neuromuscular blockade drugs
Avoidance of indirect-acting sympathomimetics
Interference with platelet function
Caution with desflurane use

A

Caution with desflurane use

Pharmacology
Patients taking chlorpromazine require special consideration with the use of desflurane during surgery due to the potential for sensitising the myocardium, which can lead to cardiac complications.

Chlorpromazine can sensitise the myocardium to the effects of desflurane, potentially leading to cardiac complications.

Avoidance of indirect-acting sympathomimetics is a concern for patients taking MAOIs, such as phenelzine or moclobemide, due to the risk of hypertensive crisis. It is not relevant for patients taking chlorpromazine.

Interference with platelet function is a concern for patients taking valproate, which can affect platelet aggregation. It is not a concern for patients taking chlorpromazine.

Prolongation of neuromuscular blockade drugs is a concern for patients taking lithium. Chlorpromazine does not significantly affect the duration of neuromuscular blockade.

Reduction in anaesthetic agent requirements is relevant for patients on lithium, as it can reduce the requirement for anaesthetic agents. It is not the primary concern for patients taking chlorpromazine.

How well did you know this?
1
Not at all
2
3
4
5
Perfectly
2
Q

A 23-year-old female is having a knee arthroscopy under general anaesthesia.

Anaesthesia is induced with fentanyl 1 mcg/kg and 2 mg/kg propofol. A supraglottic airway is inserted and anaesthesia maintained with an air oxygen mixture and 2.5% sevoflurane. The patient is allowed to breathe spontaneously through a Bain circuit. The fresh gas flow is 9 litres per minute. Over a period of 30 minutes the end-tidal CO2 rises from 4.5 kPa to 8.4 kPa. The capnograph has a baseline reading of 0 kPa.

Which one of the following is the most likely cause of the hypercarbia?

Malignant hyperthermia
Exhausted sodalime
Hypoventilation
Inadequate fresh-gas flow
Release of the tourniquet

A

Hypoventilation

Physics and clinical measurement
During anaesthesia a rise in end-tidal carbon dioxide is common in a spontaneously breathing patient. Having excluded rebreathing and increased equipment dead space, drug induced respiratory depression and hypoventilation is the most likely cause

Hypoventilation from the respiratory depressant effects of the opioid and sevoflurane are the commonest and most likely cause of a gradual rise in end-tidal CO2 (EtCO2) occurring during anaesthesia in a spontaneously breathing patient.

If the EtCO2 shows further progressive rise then other clinical signs of malignant hyperthermia should be sought.

Exhausted sodalime and inadequate fresh gas flow into the Bain circuit are causes of rebreathing and a rise in the baseline of the capnograph.

Deflation of the tourniquet would cause a sudden rise in EtCO2.

How well did you know this?
1
Not at all
2
3
4
5
Perfectly
3
Q

Which of the following does not affect resistance to flow in a blood vessel?

Haematocrit
Thickness of the vessel wall
Length of the vessel
Radius of the vessel
Pressure gradient

A

Thickness of the vessel wall

Physics and clinical measurement
Thickness of the vessel wall does not affect blood flow, but the stiffness of the vessel wall does affect blood flow.

This question refers to Poiseuille’s Law.

R = P/Q = 8nl/r4, where

P is the pressure gradient along the vessel
Q is the volume flow rate
r is the radius of the vessel
n is the viscosity (haematocrit) of the blood
l is the length of the blood vessel.

Thickness of the vessel wall does not affect blood flow, but the stiffness of the vessel wall does affect blood flow.

How well did you know this?
1
Not at all
2
3
4
5
Perfectly
4
Q

Which one of the following statements best describes a characteristic of high voltage transmission in the electrical supply system?

Efficient for long-distance transmission
Transforms DC to AC
Reduces the resistance of the conductors
Utilises chemical reactions for energy conversion
Safely used in domestic appliances

A

Efficient for long-distance transmission

High voltage transmission is a key strategy in the electrical supply system to reduce power losses over long distances. By increasing the voltage and decreasing the current, resistive losses are minimised, ensuring efficient and reliable delivery of electricity from power plants to consumers.

High voltage is used for long-distance transmission to minimise power losses due to resistance in the conductors. By increasing the voltage and reducing the current, the resistive losses (which are proportional to the square of the current) are minimised, making the transmission more efficient.

High voltage transmission does not reduce the resistance of the conductors. Instead, it reduces power loss by lowering the current for a given power level. The resistance of the conductors is a physical property that depends on the material and dimensions of the conductor.

High voltage is not used directly in domestic appliances due to safety concerns. Instead, high voltage is stepped down to safer levels using transformers before being supplied to homes and used in domestic appliances.

Transforming DC to AC is done using devices called inverters. High voltage transmission typically involves AC, not the conversion process itself. The main purpose of high voltage transmission is efficient energy transport over long distances, not transforming current types.

High voltage transmission does not involve chemical reactions. Chemical reactions are the basis of batteries and fuel cells. High voltage transmission relies on electromagnetic principles to transmit electrical energy over long distances.

How well did you know this?
1
Not at all
2
3
4
5
Perfectly
5
Q

Is it true or false that infrared analysers can be used to measure the following gases?

True / False
Oxygen
Carbon dioxide
Nitrous oxide
Nitrogen
Helium

A

False
True
True
False
False

Gases with molecules that contain at least two dissimilar atoms absorb radiation in the infrared region of the electromagnetic spectrum.

Therefore, carbon dioxide, nitrous oxide and all of the halogenated volatile anaesthetic agents can be measured using infrared absorption analysers.

Oxygen, nitrogen, helium and the inert (or noble) gases do not absorb infrared light and cannot be measured using this technology.

Oxygen is measured using the paramagnetic, galvanic or polarographic method.

How well did you know this?
1
Not at all
2
3
4
5
Perfectly
6
Q

Are the following statements true of digoxin? (High impact question)

True / False
Reduces intracellular calcium availability.
Toxicity is increased in the presence of hypokalaemia
Causes increased vagal activity.
Inhibits the cardiac sodium-potassium ATPase.
Is mainly excreted unchanged in the urine.

A

False
True
True
True
True

Digoxin has indirect effects via the vagus nerve as well as its direct effects.

Inhibition of the cardiac sodium-potassium ATPase increases the intracellular sodium concentrations and this leads to a displacement of calcium, increasing its availability. This is responsible for the positive inotropic effect.

Fifty per cent to 70% of digoxin is excreted unchanged in the urine and doses need to be altered in renal failure.

Toxicity is increased in hypokalaemia, hypomagnesaemia and hypernatraemia.

How well did you know this?
1
Not at all
2
3
4
5
Perfectly
7
Q

Are the following true or false of carbon dioxide monitoring?

True / False
Mainstream analysers cause more delay than sidestream analysers
Carbon dioxide absorbs infrared radiation at 4.28 µm
End-tidal CO2 is 0.6-0.7 kPa higher than arterial partial pressure
Is based on the principle of paramagnetism
The mainstream analyser can only be used to measure a single gas

A

False
True
False
False
False

Physics and clinical measurement
Capnography relies on the principle of infrared absorption. Carbon dioxide is made up of two dissimilar atoms and absorbs infrared radiation at a wavelength of 4.28 µm.

Capnography relies on the principle of infrared absorption. Carbon dioxide is made up of two dissimilar atoms and absorbs infrared radiation at a wavelength of 4.28 µm. End-tidal CO2 is normally 0.6-0.7 kPa lower than the arterial partial pressure.

Mainstream analyser is located within the patient’s breathing system and therefore increases dead space but allows rapid analysis. The IRMATM mainstream probes (manufactured by the company Masimo) are comercially available. They can measure CO2, N2O and five volatile anaesthetic agents (halothane, enflurane, isoflurane, sevoflurane and desflurane).

Sidestream analyser consists of a small diameter tubing that samples gases at 150-200 mL/min. It can cause sampling delay but multiple gases and anaesthetic vapours can be analysed from the same sample.

How well did you know this?
1
Not at all
2
3
4
5
Perfectly
8
Q

A patient is undergoing surgery under general anaesthesia at sea level. If the atmospheric pressure is 1 bar and the anaesthetic gas being used has a MAC of 1.5%, what is the partial pressure of the anaesthetic gas required to achieve anaesthesia?
(High impact)

0.10 bar
0.75 bar
1.5 bar
1.00 bar
0.015 bar

A

0.015 bar

Physics and clinical measurement
The Minimum Alveolar Concentration (MAC) of an anaesthetic gas is a partial pressure value that represents the potency of the anaesthetic. The MAC is expressed as a percentage of the gas in the inhaled mixture at 1 atm.

The MAC value represents the percentage of the anaesthetic gas in the inhaled mixture at 1 atm to prevent movement in 50% of patients. Therefore, for a MAC of 1.5%, the partial pressure is 1.5% of 1 atm, which is 0.015 atm.

This value does not correspond to the calculation based on the given MAC value.

This is significantly higher than what the MAC value suggests.

This would imply that the gas constitutes the entire atmospheric pressure, which is not feasible for anaesthetic gases.

This exceeds the total atmospheric pressure and is not consistent with the concept of MAC.

How well did you know this?
1
Not at all
2
3
4
5
Perfectly
9
Q

Are the following true of intravenous induction agents?

True / False
The action of propofol is terminated by metabolism.
Ketamine causes loss of consciousness in less than 30 seconds.
Etomidate is an imidazole, which is sometimes used for sedation on ITU.
Barbiturates include thiopental and methohexitone.
Propofol is more ionised than thiopentone at plasma pH.

A

False
False
False
True
False

Etomidate is indeed an imidazole but it is not used on ITU for sedation due to its depressant effect on the steroid axis. Cortisol and aldosterone synthesis is depressed for up to 24 hours after a single dose. Increased mortality is seen with its use as a sedative infusion on the ITU.

Thiopental is the new British Approved Name for thiopentone and is a thio-barbiturate, and methohexitone is an oxy-barbiturate. Both are barbiturate intravenous induction agents. Methohexitone was used for day case procedures due to its rapid onset and offset and its use has persisted for electroconvulsive therapy (ECT) due to its epileptogenic effects on the electroencephalograph (in 20% of patients). It is no longer commercially available in the UK.

Ketamine takes at least 30 seconds to cause loss of consciousness following intravenous administration (not less than 30 seconds). The rapidity of induction is related to the proportion of the drug in its unionised form.

Hence it can be seen that propofol is more unionised at plasma pH than methohexitone, which is in turn more unionised (not ionised) than thiopentone (90%, 75% and 61% respectively).

The classical induction agent the action of which is terminated by re-distribution (not metabolism) is thiopentone, but the action of propofol is similarly terminated by re-distribution.

Propofol is metabolised much more rapidly than the barbiturates and this accounts for the reduced “hangover” seen with propofol.

How well did you know this?
1
Not at all
2
3
4
5
Perfectly
10
Q

Which of the following are correct gas pipeline pressures?

True/False

4 bar for oxygen
400 kPa for power tools
7 bar for Entonox
7 bar for nitrous oxide
400 kPa for nitric oxide

A

True
False
False
False
False

A piped gas supply is made up of networks of pipes and sockets which distribute medical gases from a central source to the point of demand.

In the United Kingdom only the following are supplied by such systems:

Oxygen
Nitrous oxide (not nitric oxide)
Medical air, and
Entonox.

All pipeline gases are supplied at 4 bar (or 400 kPa), but compressed air is also supplied at 7 bar for power tools.

Carbon dioxide and nitric oxide are usually only supplied in cylinders.

How well did you know this?
1
Not at all
2
3
4
5
Perfectly
11
Q

Which one of the following best describes a primary mechanism of filtration in heat and moisture exchangers (HMEs) used in anaesthesia?
(Please select 1 option)

Thermal inactivation of pathogens by heating elements within the HME.
Electrostatic attraction capturing bacteria and viruses on filter fibres. 
Osmotic pressure gradients driving the filtration of gases and pathogens.
Chemical absorption of pathogens into the HME material's core.
Mechanical filtration through pores larger than the particles being filtered.
A

Electrostatic attraction capturing bacteria and viruses on filter fibres.

Physics and clinical measurement

A primary mechanism of filtration in heat and moisture exchangers (HMEs) is electrostatic attraction, which captures bacteria and viruses on the filter fibres, ensuring the prevention of cross-contamination.

Explanation
Electrostatic attraction capturing bacteria and viruses on filter fibres. This mechanism effectively traps pathogens on the fibres, preventing cross-contamination.

HMEs do not typically use chemical absorption to capture pathogens; their filtration relies on physical and electrostatic mechanisms.

Mechanical filtration in HMEs relies on pores smaller than the particles being filtered, not larger, to effectively trap pathogens.

HMEs do not use heating elements for thermal inactivation; they rely on passive mechanisms for filtration and moisture exchange.

Osmotic pressure gradients are not involved in the filtration mechanism of HMEs, which primarily use physical and electrostatic methods.

How well did you know this?
1
Not at all
2
3
4
5
Perfectly
12
Q

Starling forces can be applied to the pulmonary microcirculation in the same way as any other capillary bed.

Which one of the following pressures best approximates to interstitial oncotic pressure acting across a normal pulmonary capillary?

(Please select 1 option)

17 mmHg 
13 mmHg
6 mmHg 
25 mmHg
0 mmHg
A

17 mmHg

Physiology and biochemistry

The interstitial oncotic pressure is relatively high in the pulmonary circulation due to the presence of lymph.

Explanation
The typical values of the Starling forces acting across normal pulmonary capillaries are as follows:

Interstitial oncotic pressure = 17 mmHg (estimated from measurements on pulmonary lymph)

Capillary hydrostatic pressure (Pc) is 13 mmHg (arteriolar end) to 6 mmHg (venous end) but variable because of the hydrostatic effects of gravity especially in the erect lung.

Interstitial hydrostatic pressure (Pi) - Variable but ranges from zero to slightly negative.

Capillary oncotic pressure = 25 mmHg.

How well did you know this?
1
Not at all
2
3
4
5
Perfectly
13
Q

A new opioid agent with a structure similar to fentanyl, has a pKa of 6 and is metabolised in the liver by the cytochrome P450 group of enzymes. Its lipid solubility is 10:1 compared with morphine

Which single one of the following is most likely to describe the kinetic behaviour of this drug?

(Please select 1 option)

The onset of action will be slower than for alfentanil 
The clearance will be greater than that of remifentanil
The context sensitive half-time after a two-hour infusion will be shorter than that of fentanyl
The terminal elimination half-life will be shorter than that of remifentanil
The volume of distribution at steady state will be smaller than that of morphine
A

The onset of action will be slower than for alfentanil

Pharmacology

For weak acids and bases, the degree of ionization influences pharmacokinetics. The pKa of a drug relative to tissue pH influences concentration of the unionized proportion of the molecules. The greater proportion of unionised, non-polar and diffusible molecules, the faster the onset of action.

Explanation
The phenylpiperidine group of opioids are weak bases. Alfentanil has a pKa value of 6.5 and at physiological pH (7.4) approximately 90% is unionised which explains a rapid onset of action. With a pKa of 6, this new opioid will have less ionised molecules and therefore will have a slower onset of action compared with alfentanil. Remifentanil is unique because it is metabolised by non-specific esterases located in erythrocytes and other tissues. The propanoic acid-methyl ester linkage is hydrolysed. It therefore has a short terminal elimination half-life of 10-20 minutes and greater clearance because it does not depend on hepatic clearance from cytochrome enzymes.

For weak acids and bases, the degree of ionization influences pharmacokinetics. The lower pKa of this new drug (6) compared with fentanyl (8.5) means that the concentration of the unionized, diffusible form of this drug is much greater (>50%) than that of fentanyl (9%). This accounts for its more rapid onset-time and short half-life to equilibrium (t1/2 ke0).

For pharmacokinetic modelling purposes, like alfentanil this drug will have a smaller central compartment volume, a very much lower volume of distribution at steady state (Vss) and a lower clearance than fentanyl. As a result of these differences, fentanyl has a shorter context-sensitive half-time (CST) than this drug for short infusions (<2 hours). For drugs such as fentanyl, in which redistribution is the main mechanism responsible for the decline in plasma concentration after a brief infusion or bolus, the CST will initially be short. As the duration of infusion continues, redistribution becomes progressively less important and the CST increases. Therefore, fentanyl becomes a very long-acting drug if given at high infusion rates for many hours because it has a large peripheral compartment (V3) and redistribution is rapid (in contrast with propofol) and so plasma concentrations are well maintained despite rapid excretion.
Because of much greater lipid solubility, it is likely that this drug has a higher volume of distribution than morphine.

How well did you know this?
1
Not at all
2
3
4
5
Perfectly
14
Q

In the first year of life, are the following haematological features true?

True/False

The survival of erythrocytes is longer in infants than in adults
Erythropoietin production starts after 3 months Incorrect answer selected
Gamma chains are found in fetal haemoglobin (HbF)
The oxygen dissociation curve (ODC) for fetal haemoglobin lies to the right of adult haemoglobin
Sickle cell disease does not present in the neonatal period

A

False
False
True
False
True

Explanation
Fetal haemoglobin (HbF) differs from adult haemoglobin by gamma chains replacing the beta chains.

The oxygen dissociation curve for HbF is to the left of adult haemoglobin (not right), facilitating the transfer of oxygen from the maternal circulation to the fetus.

HbF is gradually replaced by adult haemoglobin during the first 6 months of life. Thus sickle cell disease does not present during the neonatal period and screening only needs to be performed after the age of 3 months (when the proportion of adult haemoglobin has started to increase).

Erythropoietin production starts in utero, and then production falls after birth with an increase seen at about 3 months resulting in a reticulocytosis.

Erythrocyte survival is shorter in neonates (not longer) than in adults.

How well did you know this?
1
Not at all
2
3
4
5
Perfectly
15
Q

Which of the following best describes the physical principles of temperature measurement by a thermocouple?

(Please select 1 option)

The bimetallic strip has a coefficient of expansion that is proportional to temperature
The bimetallic strip has a junction potential proportional to temperature 
The semiconductor at the measuring end has a junction potential that is proportional to temperature
The bimetallic strip has a resistance that is proportional to temperature
The semiconductor at the measuring end has a resistance proportional to temperature
A

The bimetallic strip has a junction potential proportional to temperature

Physics and clinical measurement

A thermocouple is constructed from a pair of dissimilar metals. The magnitude of the thermojunction electromotive force at the measuring end is proportional to applied temperature.

Explanation
A thermocouple is a device that is used to measure temperature “electronically”. It consists of a pair of dissimilar metal (bimetallic) wires or strips bonded together, typically copper and constantan (an alloy of 55% copper and 45% nickel). When these wires contact each other a small voltage is generated in the order of millivolts. The magnitude of the thermojunction electromotive force (emf) is proportional to applied temperature (the Seebeck effect). This physical principle can be applied to the measurement of temperature. The electromotive force at the measuring junction is proportional to temperature.

Two wires bonded together with different coefficients of expansion can be used as a switch for thermostatic control.

The resistance of the measuring junction of a thermocouple is irrelevant. The thermocouple does not employ semiconductor materials for temperature measurement.

How well did you know this?
1
Not at all
2
3
4
5
Perfectly
16
Q

Iron is an essential micronutrient in the human body.

Which one of the following options is the most abundant store of iron in the body?

(Please select 1 option)

Ferritin and haemosiderin
Haem enzymes
Haemoglobin 
Transferrin bound iron
Myoglobin
A

Haemoglobin

Key Learning Points
Physiology and biochemistry

Haemoglobin is the most abundant store of iron in the body

Explanation
Iron is an essential micronutrient, as it is required for adequate erythropoietic function, oxidative metabolism and cellular immune responses. Although the absorption of dietary iron (1-2 mg/d) is regulated tightly, it is just balanced with losses.

There are 35-45 mg/kg iron in the adult body (about 4-5 g)

Iron is present in the following forms:

Haemoglobin 65%
Ferritin and haemosiderin 30%
Myoglobin 3.5%
Haem enzymes 0.5%
Transferrin bound proteins 0.1%

How well did you know this?
1
Not at all
2
3
4
5
Perfectly
17
Q

With regard to sodalime, which one of the following statements is correct?

(Please select 1 option)

The absorption of CO2 is an endothermic reaction
The main component is calcium carbonate
4-8mesh is a metric measurement of granule size
100 grams of soda lime can absorb up to 26 litres of CO2 
Passing sevoflurane through dry sodalime can generate clinically significant amounts of carbon monoxide (CO)
A

100 grams of soda lime can absorb up to 26 litres of CO2

Key Learning Points
Physics and clinical measurement

The main components of sodalime depend on the manufacturer.

Explanation
The main components of sodalime depend on the manufacturer. Typically it will be composed of:

Calcium hydroxide (93-97%)
Sodium hydroxide (0-3%)
Potassium hydroxide (0-1%)
Hardeners - Silicates (0-7.5%), Zeolite (0-5%)
Indicator (0.03%)
Water (13-17%)
Size of granules 4-8 mesh

First step:
Carbon dioxide and the water contained in the soda lime react to form carbonic acid.
CO2 + H2O = H2CO3

Intermediate step:
Carbonic acid reacts exothermically with sodium hydroxide to form sodium carbonate and water.
H2CO3 + 2 NaOH = Na2CO3 + 2 H2O + heat

Final step:
Sodium carbonate reacts with the slaked lime to form calcium carbonate and sodium hydroxide: Na2CO3 + Ca(OH)2 = CaCO3 + 2 NaOH
100 grams of soda lime can absorb up to 26 litres of CO2.

Desflurane and enflurane have been reported to produce substantial amounts of carbon monoxide (CO) when passed through dry sodalime. Isoflurane is said to produce less CO. Sevoflurane and halothane produce very little CO.

The granule size range of 4 to 8 mesh is an imperial measurement (inches) allows high absorptive capacity at low resistance to gas flow.

How well did you know this?
1
Not at all
2
3
4
5
Perfectly
18
Q

Typical features of aspirin overdose include:

True/False

Hyperventilation
Tinnitus
Hypothermia
Respiratory acidosis
Hyperkalaemia

A

True
True
False
False
False

Explanation
Severe aspirin toxicity occurs with serum concentrations >750 mg/L.

It may present with:

Tinnitus
Deafness
Pyrexia
Hypoglycaemia
Haematemesis
Hyperventilation, and
Hypokalaemia

Aspirin overdose typically produces a respiratory alkalosis due to a direct stimulatory effect on the respiratory centre. To compensate for this there is a loss of bicarbonate from the urine with loss of sodium, potassium and water.

Aspirin and NSAIDs can cause hyperkalaemia by inducing a hyporenin hypoaldosteronemic state caused by inhibition of prostaglandin synthesis but this is not typical, especially if the patient is euvolaemic and has normal renal function.

How well did you know this?
1
Not at all
2
3
4
5
Perfectly
19
Q

Do the following drugs inhibit enzymes?

True/False

Piroxicam
Allopurinol
Trandolapril
Acetazolamide
Cimetidine

A

True
True
True
True
True

Explanation
Acetazolamide is a carbonic anhydrase inhibitor and a weak diuretic. It is used for prophylaxis against mountain sickness and in the management of glaucoma.

Allopurinol inhibits the enzyme xanthine oxidase which is responsible for urate synthesis and is used in the management of gout.

Piroxicam is a non-steroidal anti-inflammatory drug (NSAID) that inhibits cyclo-oxygenase.

Trandolapril is an angiotensin-converting enzyme inhibitor (ACE inhibitor).

Cimetidine is a histamine type-2 receptor antagonist but as a side effect it inhibits the mixed function oxidases (cytochrome P-450s) and so retards oxidative hepatic drug metabolism (phase 1 reactions).

How well did you know this?
1
Not at all
2
3
4
5
Perfectly
20
Q

Are the following statements true or false?

True/False

100 ml of arterial blood carries 47.5 ml CO2 in the form of bicarbonate
The chloride shift is the movement of chloride ions from red blood cells into the plasma
The Haldane effect describes the reduced buffering capacity of haemoglobin once deoxygenated
The Anrep effect relates to the heart rate response to hypoxia
The Bohr effect relates to increased carbon dioxide carriage by deoxygenated blood

A

False
False
False
False
False

In the red blood cell carbon dioxide is converted by carbonic anhydrase to carbonic acid, which in turn dissociates to hydrogen and bicarbonate ions.

The chloride shift (or Hamburger shift) is the movement of chloride ions into red blood cells (not plasma) as bicarbonate ions enter the plasma, which maintain electrical neutrality. The hydrogen ions are buffered mainly by haemoglobin.

The Haldane effect refers to haemoglobin’s increased buffering ability as it becomes deoxygenated.

The Bohr effect is the rightward shift of the oxyhaemoglobin dissociation curve associated with a rise in arterial PCO2.

In arterial blood approximately 50 ml of carbon dioxide is carried per 100 ml of blood: 45 ml as bicarbonate, 2.5 ml as carbonic acid and 2.5 ml as carbamino compounds. Venous blood carries 54 ml of carbon dioxide and 47.5 ml of this is as bicarbonate.

The Anrep effect describes the intrinsic regulatory mechanism of the heart in response to an increased afterload.

How well did you know this?
1
Not at all
2
3
4
5
Perfectly
21
Q

Cellular respiration involves an energy substrate and oxygen with the production of carbon dioxide and water. The respiratory quotient for a particular patient is measured as 0.7.

Which of the following is the most likely combination of energy substrates of this patient’s diet?

(Please select 1 option)

High carbohydrate, high fat and low protein
Low carbohydrate, high fat and low protein 
Low carbohydrate, low fat and high protein
High carbohydrate, high fat and high protein
High carbohydrate, low fat and low protein
A

Low carbohydrate, high fat and low protein

Key Learning Points
Physiology and biochemistry

The RQ for a diet that is predominantly fat is 0.7. Lipids require more oxygen than carbohydrates for complete oxidation.

The Respiratory quotient (RQ) is the ratio of CO2 produced by the body to the volume of O2 consumed per unit time.

RQ = CO2 produced / O2 consumed.

Typically 200 mL/minute CO2 produced and 250 mL/minute O2 consumed. A mixed diet will typically produce and RQ of about 0.8.

The RQ will vary with the energy substrates in the diet.

Granulated sugar is a pure refined carbohydrate and is 99.999% carbohydrate with no significant lipid, proteins, minerals or vitamin content.

Glucose and other hexose sugars - RQ = 1

Fats - RQ = 0.7 (lipids require more oxygen than carbohydrates for complete oxidation)
Proteins - RQ is approximately 0.8-0.9
Ethyl alcohol - RQ = 0.67

How well did you know this?
1
Not at all
2
3
4
5
Perfectly
22
Q

Pure water at pH 7 has hydrogen ion concentration of:

(Please select 1 option)

40 nanomol/L
1000 nanomol/L
100 nanomol/L
70 nanomol/L
0 nanomol/L
A

100 nanomol/L

Key Learning Points
Physics and clinical measurement

pH is the negative log to the base 10 of hydrogen ion concentration. The relationship between hydrogen ion concentration and pH is exponential.

pH is the negative log to the base 10 of hydrogen ion concentration:

Logarithms
Ask the question what power produces the answer

pH = - log10 [H+], rearranging the equation:

[H+] = 10-pH or substituting pH, [H+] = 10-7

One nanomol = 1 x 10-9 or 0.000000001

10-7 = 1x 0.0000001 or 10 x 0.00000001 or 100 x 0.000000001

100 nanomol

How well did you know this?
1
Not at all
2
3
4
5
Perfectly
23
Q

Do the following respiratory changes occur in a normal pregnancy?

True/False

Respiratory alkalosis
Tidal volume increases
Functional residual capacity is increased
Lung compliance is reduced
Plasma bicarbonate is reduced

A

True
True
False
False
True

There is a reduction in total respiratory compliance due to a decrease in chest wall compliance, but lung compliance is not altered.

The functional residual capacity (FRC) is reduced by 20% (not increased), which is mainly due to elevation of the diaphragm.

Progesterone stimulates the respiratory centre producing primarily an increase in tidal volume with a relatively constant respiratory rate, this leads to a respiratory alkalosis which is compensated for by a reduction in plasma bicarbonate levels.

How well did you know this?
1
Not at all
2
3
4
5
Perfectly
24
Q

A 30-year-old woman presents with vertical diplopia and difficulty reading and walking downstairs. On examination, her left eye appears slightly elevated and adducted. She reports that tilting her head to the right improves her vision.

Which one of the following cranial nerve palsies is most likely?
(Please select 1 option)

Abducens nerve palsy 
Optic nerve palsy
Trochlear nerve palsy 
Trigeminal nerve palsy
Oculomotor nerve palsy
A

Trochlear nerve palsy

Key Learning Points
Anatomy

Trochlear nerve palsy leads to vertical diplopia and patients often adopt a head tilt to compensate. Recognising the clinical presentation helps in diagnosing and managing cranial nerve palsies effectively.

Trochlear nerve palsy: This presents with vertical diplopia and compensatory head tilt (to the side opposite the lesion) to align the vision, as the superior oblique muscle is affected, causing the eye to be elevated and adducted.

Abducens nerve palsy: This would cause horizontal diplopia and difficulty abducting the eye but would not typically cause vertical diplopia or issues with head tilt.

Oculomotor nerve palsy: This would cause the eye to be down and out with ptosis, rather than the symptoms described.

Optic nerve palsy: This would result in visual loss, not diplopia or issues with head tilt.

Trigeminal nerve palsy: This affects facial sensation and mastication muscles, not eye movement.

How well did you know this?
1
Not at all
2
3
4
5
Perfectly
24
Q

Which one of the following is the primary role of a capacitor in an AC circuit?
(Please select 1 option)

To measure the frequency of the AC signal
To convert electrical energy into heat
To provide a steady output voltage
To block direct current while allowing alternating current to pass 
To act as a power source
A

To block direct current while allowing alternating current to pass

Key Learning Points
Physics and clinical measurement

In AC circuits, capacitors are used to block direct current while allowing alternating current to pass through. This characteristic makes them essential in applications like coupling and decoupling in signal processing and filtering unwanted noise from signals.

Explanation
Capacitors block direct current (DC) because they cannot maintain a constant flow of charge, but they allow alternating current (AC) to pass through because they can continuously charge and discharge in response to the changing voltage.

Power sources provide the necessary energy to power a circuit, such as batteries or power supplies. Capacitors store and release electrical energy but do not generate it.

Capacitors can smooth voltage fluctuations but do not regulate voltage on their own.

Measuring the frequency of an AC signal is typically done with frequency counters or oscilloscopes. Capacitors do not have the capability to measure frequency.

Capacitors store and release electrical energy but do not convert it into heat as their primary function.

How well did you know this?
1
Not at all
2
3
4
5
Perfectly
24
Q

Are the following true regarding labetalol?

True/False

Has alpha blocking action
Decreases bile secretion
Has a half life of two hours
Causes bronchodilation
Is 70% protein bound

A

Causes bronchodilation

Explanation
Labetalol is a combined beta and alpha adrenergic receptor antagonist with a ratio of activity between 2:1 and 5:1 respectively.

It is selective for alpha-1 receptors but is non-selective for beta receptors.

It is used to treat severe hypertension and pre-eclampsia and in hypotensive anaesthesia.

Labetalol has a half life of approximately four hours (not two) and is approximately 50% protein bound.

It has been shown to cross the placental barrier, but not the blood brain barrier. It is metabolised in the liver and excreted in the urine and faeces.

Oral administration of the drug undergoes extensive first-pass metabolism.

Severe hepatocellular damage has been reported after both short and long term use and the reduction of bile secretion may rarely lead to jaundice.

How well did you know this?
1
Not at all
2
3
4
5
Perfectly
25
Q

Which one of the following is the primary role of a capacitor in an AC circuit?
(Please select 1 option)

To block direct current while allowing alternating current to pass 
To provide a steady output voltage
To convert electrical energy into heat
To measure the frequency of the AC signal
To act as a power source
A

To block direct current while allowing alternating current to pass

Key Learning Points
Physics and clinical measurement

In AC circuits, capacitors are used to block direct current while allowing alternating current to pass through. This characteristic makes them essential in applications like coupling and decoupling in signal processing and filtering unwanted noise from signals.

Explanation
Capacitors block direct current (DC) because they cannot maintain a constant flow of charge, but they allow alternating current (AC) to pass through because they can continuously charge and discharge in response to the changing voltage.

Power sources provide the necessary energy to power a circuit, such as batteries or power supplies. Capacitors store and release electrical energy but do not generate it.

Capacitors can smooth voltage fluctuations but do not regulate voltage on their own.

Measuring the frequency of an AC signal is typically done with frequency counters or oscilloscopes. Capacitors do not have the capability to measure frequency.

Capacitors store and release electrical energy but do not convert it into heat as their primary function.

How well did you know this?
1
Not at all
2
3
4
5
Perfectly
26
Q

In a clinical trial of a new intravenous induction agent, the plasma concentrations of this drug are taken at regular time intervals following a bolus.

The following data are acquired:

Time following injection (hours) Plasma concentration (mcg/mL)
2 400
6 100
10 25
14 6.25

From these measurements which one of the following options approximates best to the plasma half life (T½) of the drug?

(Please select 1 option)

4 hours
1 hour
10 hours
2 hours 
8 hours
A

2 hours

Key Learning Points
Pharmacology

The plasma half life (T½) of a drug is the time it takes for the initial plasma concentration to fall by 50%.

Explanation
The plasma half life (T½) of a drug is the time it takes for the initial plasma concentration to fall by 50%.

Once injected into the central compartment (intravascular space) the concentration of the intravenous induction agent decreases exponentially as it is eliminated and redistributed.

Extrapolating the values from the plasma concentration vs. time data:

Plasma concentration at time zero = 800 mcg/mL
Plasma concentration at 2 hours = 400 mcg/mL
Plasma concentration at 4 hours = 200 mcg/mL
Plasma concentration at 6 hours = 100 mcg/mL
Plasma concentration at 8 hours = 50 mcg/mL

How well did you know this?
1
Not at all
2
3
4
5
Perfectly
27
Q

Concerning the femoral nerve and its branches, which statement is correct?

(Please select 1 option)

Lies within the femoral sheath
A branch supplies the skin of part of the foot 
Is formed from the dorsal divisions of the L1-L4 anterior rami of the spinal cord 
Lies medial to the femoral artery in the femoral triangle
Has a branch which supplies the skin of the scrotum
A

A branch supplies the skin of part of the foot

Key Learning Points
Anatomy

The femoral nerve is formed from the dorsal divisions of the L2-L4 ventral rami of the spinal cord and runs down the medial side of the leg, supplying the medial side of the calf and the medial side of the dorsum of the foot, before terminating in the region of the ball of the big toe.

Explanation
The saphenous nerve, a branch of the femoral nerve, runs down the medial side of the leg and supplies the medial side of the calf as far as the medial malleolus. It terminates in the region of the ball of the big toe and may supply the medial side of the dorsum of the foot. The scrotum derives its sensory supply from the pudendal, posterior femoral cutaneous, genitofemoral and ilioinguinal nerves. In the femoral triangle the femoral nerve is lateral to the artery which in turn is lateral to the femoral vein. It lies outside the femoral sheath which “houses” the femoral artery and vein, lymph nodes, femoral canal and loose connective tissue. The femoral nerve is s formed from the dorsal divisions of the L2-L4 ventral rami of the spinal cord.

How well did you know this?
1
Not at all
2
3
4
5
Perfectly
28
Q

Which of the one of the following statements about the adult human trachea best applies?

It is a midline structure that bifurcates at T4
The right main bronchus angles off the trachea at 45°
It is a midline structure that starts at C4
The left main bronchus angles off the trachea at 25°
It is approximately 25 cm long incorrect option

A

It is a midline structure that bifurcates at T4

Key learning points
Anatomy

The human trachea is approximately 15 cm long and bifurcates at the level of T4, with the right main bronchus separating at a 25° angle and the left main bronchus separating at a 45° angle.

Explanation

The human trachea starts at C6 at the inferior edge of the cricoid cartilage and extends to the level of T4 where the trachea bifurcates. It is approximately 15 cm long in an adult. The right main bronchus separates from the trachea at a 25° angle, whereas the left main bronchus separates at an angle of 45°.

How well did you know this?
1
Not at all
2
3
4
5
Perfectly
29
Q

Is it true or false that the approximate pH of intravenous fluids is as follows?

True / False
3% hypertonic saline is 7.4
Hartmann’s solution is 6.5
0.9% sodium chloride is 5.5
5% glucose is 6.8
Gelofusine is 7.4

A

False
True
True
False
True

Explanation

The pH values of intravenous fluids may vary slightly between the different manufacturers, but approximate pH values (+/- 0.3) are as follows:

Sodium chloride 0.9% is 5.5
Hartmann’s is 6.5
Glucose 5% is 4.15
Gelofusine is 7.4
3% hypertonic saline is 5.0

How well did you know this?
1
Not at all
2
3
4
5
Perfectly
30
Q

Are the following statements true or false about volatile agents?

True/False

Desflurane has a lower boiling point than halothane
Sevoflurane has a higher minimal alveolar concentration (MAC) than desflurane
Isoflurane and enflurane have the same minimal alveolar concentration (MAC)
Halothane and isoflurane have a similar saturated vapour pressure (SVP)
Sevoflurane has a higher molecular weight than desflurane

A

True
False
False
True
True

Isoflurane and enflurane are structural isomers and have the same molecular weights, but different minimal alveolar concentration (MAC) values.

Sevoflurane has a molecular weight of 200, desflurane has a molecular weight of 168. The MAC values of sevoflurane and desflurane are 1.8 and 6 respectively.

At 23.5°C, desflurane has the lowest boiling point of all the volatile agents in common use, and requires a specialised vapouriser.

Halothane has a boiling point of 50°C.

Saturated vapour pressure (SVP) is an indicator of volatility and is measured in kPa. The SVP is similar for both halothane and isoflurane at approximately 32kPa (240 mmHg).

How well did you know this?
1
Not at all
2
3
4
5
Perfectly
31
Q

The most appropriate method of confirming the correct position of a new tracheostomy tube is:
(Please select 1 option)

Chest X-ray
Ultrasound
Capnography I
Physical examination
Endoscopy via the tracheostomy tube
A

Endoscopy via the tracheostomy tube

Key Learning Points
Physiology and biochemistry

Endoscopy via the tracheostomy tube is the most accurate method to confirm the correct positioning of the tube within the trachea.

Explanation
Endoscopy provides a detailed assessment of the tube tip position in relation to the carina and within the tracheal lumen.

Chest X-ray can confirm tube placement but lacks detail on exact positioning.

Capnography confirms the tube is in the trachea but not its precise location.

Physical examination is not reliable for assessing internal tube position.

Ultrasound can be useful but doesn’t provide as detailed information as endoscopy.

32
Q

During an advanced life support (ALS) scenario, a 55-year-old man presents with a non-shockable rhythm of pulseless electrical activity (PEA) during cardiac arrest.

Under these circumstances, how should adrenaline be administered?

1 mg IV/IO after the third defibrillation attempt
500 mcg IV/IO every 3-5 minutes while ALS continues
300 mcg IV/IO immediately, followed by 1 mg IV/IO after 10 minutes
1 mg IV/IO as soon as the non-shockable rhythm is confirmed
1 mg IV/IO after the first defibrillation attempt

A

1 mg IV/IO as soon as the non-shockable rhythm is confirmed

Key learning points
Other
In adult cardiac arrest with a non-shockable rhythm such as PEA or asystole, 1 mg of adrenaline IV/IO should be administered as soon as possible and repeated every 3-5 minutes during ongoing resuscitation efforts.

Explanation

1 mg IV/IO as soon as the non-shockable rhythm is confirmed This is the correct answer. In cases of non-shockable rhythms such as PEA, immediate administration of adrenaline is recommended to maximise the chances of return of spontaneous circulation by enhancing myocardial and cerebral blood flow.

1 mg IV/IO after the first defibrillation attempt is incorrect as defibrillation attempts are not applicable in non-shockable rhythms like PEA. Adrenaline should be administered as soon as a non-shockable rhythm is confirmed to support coronary and cerebral perfusion.

500 mcg IV/IO every 3-5 minutes while ALS continues is incorrect as the recommended dose of adrenaline during cardiac arrest in adults is 1 mg IV/IO. A dose of 500 mcg is lower than the guideline recommendation and may be insufficient during ALS.

1 mg IV/IO after the third defibrillation attempt is inappropriate for a non-shockable rhythm scenario. This option refers to the administration of adrenaline in shockable rhythms like ventricular fibrillation or pulseless ventricular tachycardia after three unsuccessful shocks.

300 mcg IV/IO immediately, followed by 1 mg IV/IO after 10 minutes does not align with current guidelines. The initial recommended dose for an adult in cardiac arrest is 1 mg of adrenaline, and this dosing strategy could delay necessary intervention.

33
Q

Which of the following is more specific to bipolar compared with unipolar diathermy?

Has a power output of up to 140 joules per second
Has a high current density causing tissue damage
May interfere with pacemaker function
Has a frequency of 500 KHz - 1 MHz
Requires an isolating capacitor

A

Has a power output of up to 140 joules per second

Key learning points
Physics and clinical measurement
Bipolar diathermy has a lower power output compared with unipolar diathermy.

Explanation

Electrocautery or diathermy uses high frequency current to generate heat to enable coagulation, cutting of tissue or fulguration (cell destruction from dehydration).

With bipolar diathermy the two electrodes are the tips of forceps and current passes between the tips and not through the patient. The power output of bipolar diathermy (40-140 W) is less than the typical output of unipolar diathermy (400 W). The bipolar circuit is not earthed.

Unipolar diathermy has a cutting electrode and an “indifferent” electrode in the form of a metal plate. The high frequency current passes through the patient from the active electrode to the metal plate to complete a circuit. The current density at the indifferent electrode is low when correctly applied and unlikely to burn the patient. An isolating capacitor is placed between the patient plate and the earth. This is incorporated into modern diathermy machines and has a low impedance to a high frequency current, that is, diathermy current. The capacitor has high impedance to 50 Hz current, protecting the patient against electrical shock.

Both unipolar and bipolar diathermy use high frequency currents (500 KHz - 1 MHz) sufficient to cause tissue damage and have the potential to interfere with pacemaker function (less so with bipolar diathermy).

The effect of diathermy depends upon current density and the waveform used. In coagulation mode the current is a pulsed square wave and in cutting mode the current is a continuous square wave pattern.

34
Q

Which one of the following drugs when given by infusion is associated with a reflex tachycardia?

Dexmedetomidine
Phentolamine
Clonidine
Methyldopa
Tizanidine

A

Phentolamine

Key learning points
Pharmacology
Alpha-1 and alpha-2 adrenoreceptor blockade is associated with the development of a reflex tachycardia. This may develop with phentolamine or phenoxybenzamine.
Explanation

There are three subtypes of α-2 adrenoceptors (2a, 2b and 2c). The receptors are usually presynaptic, where they inhibit the release of noradrenaline at nerve endings. The α-2 agonists affect receptors in both the central and peripheral nervous systems. Centrally in the locus coeruleus (in the brainstem), α-2 agonists produce sedation, analgesia, and euphoric effects, reduce the MAC of volatile anaesthetic agents and used to treat acute withdrawal symptoms of chronic opioid users.

The most likely effect on the heart rate from the α-2 agonists is bradycardia. Phenoxybenzamine is an α-1 and α-2 adrenoreceptor antagonist.

Clonidine is a selective α-2 agonist, with an affinity ratio of 200:1 for α-2: α-1 receptors, respectively.

Tizanidine is a drug similar to clonidine, but with some significant differences. Like clonidine, it has sedative, anxiolytic, and analgesic properties, but it has a shorter duration of action and less effect on heart rate and blood pressure.

Dexmedetomidine is a highly selective α-2 adrenoreceptor agonist similar to clonidine but with a greater affinity for the α-2 receptor. The affinity ratio is 1620 :1 for α-2 : α-1 receptors, respectively. It has a biphasic effect on the blood pressure. Initially, it causes a transient increase in blood pressure with a reflex bradycardia (stimulation of α-2b subtypes of receptors present in vascular smooth muscles) followed by a decrease in sympathetic outflow from the brainstem and hypotension/bradycardia.

Methyldopa is a prodrug. It inhibits dopa-decarboxylase that catalyses the conversion of L-dopa to dopamine (a precursor of noradrenaline and adrenaline). It is also metabolised to alpha-methylnoradrenaline, a centrally acting α-2 adrenoreceptor agonist. These two mechanisms contribute to its effects on reducing blood pressure. Cardiac output usually is maintained without an increase in heart rate. In some patients the heart rate is slowed.

Phentolamine is a short-acting antagonist to peripheral α-1 and α-2 receptors, resulting in a decrease in peripheral vascular resistance and vasodilatation. It is used in the treatment of acute hypertensive emergencies (e.g. hypertension from pheochromocytoma). A drop in systemic vascular resistance often leads to a reflex tachycardia via a baroreceptor reflex

35
Q

A fit 30-year-old man attends an outpatient clinic for a blood test. He subsequently “faints” at the sight of the needle.

Which of the following receptors is primarily responsible for his “faint”?

Post-ganglionic muscarinic receptors in the heart
Pre-ganglionic nicotinic receptors within the parasympathetic ganglion
Muscarinic receptors in the nucleus accumbens
Nicotinic receptors at the skeletal neuromuscular junction
Nicotinic receptors in the tractus solitarius

A

Post-ganglionic muscarinic receptors in the heart

Key learning points
Physiology and biochemistry
Discharge from afferent vagal cardiac C fibres overcome the influence of sympathetic activity causing neurocardiogenic syncope.

Explanation

The symptoms suggest this patient is experiencing neurocardiogenic or vasovagal syncope. This is usually a benign condition characterised by a self-limited episode of systemic hypotension and a transient loss of consciousness or “faint”.

Higher cortical centres in response to a triggering event (e.g. panic, fright or pain) stimulate adrenergic tone. This results in a tachycardia and increased myocardial contractility. Mechanoreceptors in the left ventricle are not only stimulated by stretch but also by changes in systolic contraction.

Stimulation of mechanosensitive afferent vagal cardiac C fibres results in vasodilatation and an increase in vagal tone with a reduction in cardiac filling and profound bradycardia. The receptors innervated by the vagus in the sinoatrial node are post-ganglionic muscarinic (M2) receptors.

The nucleus tractus solitarius (NTS) is a primary integrative centre for cardiovascular control and other autonomic functions within the central nervous system. Baroreceptor afferent messages are first integrated within the NTS and it is thought that an excitatory amino acid (glutamate) is the principal neurotransmitter of corresponding afferents fibres. Evidence points to the fact that 5-HT acts at 5-HT2A receptors, facilitates the baroreceptor reflex.

The nucleus accumbens is part of the mesolimbic region of the brain that receives dopaminergic projections from the brainstem and influences reward behaviour, and it is thought to be primarily involved in reinforcing addictive behaviour in response to drug use.

36
Q

During the resuscitation of a neonate, if there is no response to CPR and emergency vascular access is established, what is the initial dose of adrenaline recommended?

30 micrograms kg^-1
50 micrograms kg^-1
10 micrograms kg^-1
100 micrograms kg^-1
20 micrograms kg^-1

A

20 micrograms kg^-1

Key learning points
Other
The recommended initial dose of adrenaline during neonatal resuscitation is 20 micrograms kg^-1 (0.2 mL kg^-1 of 1:10,000 adrenaline), which is administered to improve myocardial and cerebral perfusion when response to CPR is absent. Correct dosing is crucial to optimise outcomes while minimising potential risks.

Explanation

20 micrograms kg^-1:
This is the recommended initial dose of adrenaline (0.2 mL kg^-1 of 1:10,000 adrenaline) in neonatal resuscitation. It is effective in increasing heart rate and blood pressure, thereby improving coronary and cerebral perfusion during CPR.

10 micrograms kg^-1:
This dose is lower than the recommended initial dose for neonatal resuscitation. While providing some adrenaline effect, it may be insufficient in reversing the effects of severe bradycardia or asystole in a neonatal cardiac arrest scenario.

30 micrograms kg^-1:
Although a higher dose of adrenaline could theoretically enhance the cardiovascular response, it can also increase the risk of adverse effects such as excessive vasoconstriction and possible subsequent myocardial ischemia.

0 micrograms kg^-1:
This dose significantly exceeds the recommended initial dose for neonatal resuscitation and may lead to harmful effects, including increased myocardial oxygen demand and potential arrhythmias.

100 micrograms kg^-1:
This dose is excessively high for initial resuscitation efforts and is associated with a high risk of adverse reactions, such as hypertension, tachycardia, and arrhythmias, which can be detrimental in the critical phase of neonatal resuscitation.

37
Q

A dominant R wave in lead V1 is characteristic in:

True / False
LBBB
Pulmonary embolus
Hyperkalaemia
WPW
True posterior myocardial infarction

A

False
False
False
True
True

Explanation

A rightward shift of axis acutely can produce a dominant R wave in lead V1 in a massive PE but ‘characteristic’ implies commonly seen, which it is not.

It is characteristic in Wolff-Parkinson-White (WPW) syndrome type A.

In LBBB and hyperkalaemia a leftward shift is commonly seen.

38
Q

Are the following true or false with regards to mast cells?

True / False
Are lipophilic cells involved in inflammatory and immune responses
An excess of circulating mast cells causes mastocytosis
Contain heparin
Degranulation releases lytic enzymes and inflammatory mediators from storage granules
Cross-linkage of surface IgA molecules by antigen may cause an anaphylactic reaction

A

False
True
True
True
False

Key learning points
Physiology and biochemistry

Mast cells are basophilic cells (not lipophilic) in the connective and subcutaneous tissues, which are involved in inflammatory and immune responses.

Explanation

Mast cells are basophilic cells (not lipophilic) in the connective and subcutaneous tissues, which are involved in inflammatory and immune responses.

They contain storage granules that contain lytic enzymes, for example, tryptase, and inflammatory mediators, for example, histamine, heparin, 5-HT, leukotrienes, platelet aggregating factor, leucocyte chemotactic factor and hyaluronidase.

Release of these mediators occurs during mast cell degranulation, which can be triggered by:

tissue injury
drugs
complement activation, and
foreign antigenic material.
An anaphylactic reaction occurs when a previously sensitised individual is re-exposed to the antigen. It is an IgE mediated immune response (not IgA).

Mastocytosis occurs when excess mast cells are present in the circulation or as tissue infiltrates.

Diagnosis is based on seum tryptase and bone marrow findings

39
Q

A 20-year-old male is admitted to the Emergency department with a diagnosis of status epilepticus.

The paramedics have already administered high flow oxygen and 10 mg rectal diazepam. The vital signs are a pulse rate of 140 beats per minute; respiratory rate 25 breaths per minute; blood pressure 130/80 mmHg; oxygen saturation 98% and a patent airway but continued tonic-clonic seizures. His blood glucose is 7.0 mmol/L.

Which of the following agents is the most appropriate initial pharmacological treatment?

Propofol
Thiopentone
Phenobarbitone
Phenytoin
Lorazepam

A

Lorazepam

Key learning points
Pharmacology
Lorazepam is the first line benzodiazepine in the hospital setting. Diazepam should be given if lorazepam is not available. Familiarise yourself with the treatment algorithm of status epilepticus.

Explanation

Lorazepam has emerged as the preferred benzodiazepine for acute management of status epilepticus.

Lorazepam differs from diazepam in two important respects. It is less lipid-soluble than diazepam, with a distribution half life of two to three hours versus 15 minutes for diazepam. Therefore, it should have a longer duration of clinical effect. Lorazepam also binds the GABAergic receptor more tightly than diazepam, resulting in a longer duration of action. The intravenous dose is 0.1 mg/kg. The second line drug is intravenous phenytoin 15-20 mg/kg.

Status epilepticus is defined as convulsions lasting for 5 or more minutes or recurrent episodes of convulsions in a 5-minute period without return to pre-convulsive neurological baseline. A typical secondarily generalize tonic-clonic seizure generally stops by three minutes and almost always by 5-minutes. Status epilepticus was previously defined as continuous seizure activity of at least 30 minutes duration or intermittent seizure activity of at least 30 minutes duration during which consciousness is not regained.

Phenytoin is one of the most effective drugs for treating acute seizures and status epilepticus. The main advantage of phenytoin is the lack of a sedating effect. However, a number of potentially serious adverse effects may occur. Arrhythmias and hypotension have been reported. The side effects of phenytoin have largely been attributed to its diluent propylene glycol or rapid administration. Fosphenytoin (a prodrug) is more cardiovascularly stable.

Phenobarbital typically is used after a benzodiazepine or phenytoin has failed to control status epilepticus. The normal loading dose is 15 to 20 mg per kg. Because high-dose phenobarbital is sedating, airway protection is an important consideration.

Where airway protection or control ventilation is required, then the drugs of choice are intravenous thiopentone or propofol.

40
Q

Which of the following statements correctly describes one aspect of the innervation of the human larynx?

Total bilateral recurrent laryngeal nerve palsy leads to severe abductor spasm
The recurrent laryngeal nerves are sensory to the laryngeal mucous membrane below the level of the vocal cords
In the cadaveric position the cords are fully abducted
The glossopharyngeal nerves are sensory to the laryngeal mucous membrane above the level of the vocal cords
The internal laryngeal nerve supplies the lingual surface of the epiglottis

A

The recurrent laryngeal nerves are sensory to the laryngeal mucous membrane below the level of the vocal cords

Key learning points
Physics and clinical measurement
The recurrent laryngeal nerves provide sensory innervation to the laryngeal mucous membrane below the level of the vocal cords. The glottis, supraglottis and inferior epiglottis are supplied by the internal laryngeal nerve and the lingual surface of the epiglottis supplied by the glossopharyngeal nerve.

Explanation

The superior laryngeal nerve is a branch of the vagus (CNX). This divides into the internal and external laryngeal nerves. The external laryngeal nerve supplies the cricothyroid muscle of the larynx, which tenses the vocal folds. The internal laryngeal provides sensory innervation to the glottis, supraglottis and inferior epiglottis.

The lingual surface of the epiglottis is supplied by the glossopharyngeal nerve (CN1X).

In the cadaveric position the vocal cords are in an intermediate position. During inspiration and expiration the vocal cords are fully abducted.

The recurrent laryngeal nerve is a branch of the vagus nerve and supplies all the intrinsic muscle of the larynx with the exception of the cricothyroid muscles. It provides the sensory supply to the larynx below the vocal cords. Bilateral transection of recurrent laryngeal nerves will result in the inability of the posterior cricoarytenoid muscles to abduct the vocal cords resulting in stridor.

The glossopharyngeal nerve provides sensory innervation to the posterior third of the tongue, pharynx, carotid sinus and carotid body. It has motor fibres to the stylopharyngeal muscle and provides parasympathetic supply to the parotid gland.

41
Q

A 2-year-old child presents for inguinal hernia repair. A caudal anaesthetic block is planned for this procedure.

Which one of the following options best explains why access to the caudal epidural space is via the sacral hiatus?

The sacrococcygeal membrane is tough and offers noticeable resistance
The dural sac ends at the level of the sacral hiatus
The failure of fusion of the laminae of S4 and S5 provides a suitable point of entry
The sacral hiatus is palpable only in the lateral position
The posterior superior iliac spines and sacral hiatus form an equilateral triangle pointing downwards

A

The failure of fusion of the laminae of S4 and S5 provides a suitable point of entry

Key learning points
Anatomy
The sacral canal results from the fusion of the laminae of the five sacral vertebrae in the midline. Failure of fusion of the S4 and S5 results in the formation of the sacral hiatus. The sacral hiatus is easily identifiable as a small depression between the sacral cornua, hence making the performance of the sacral block possible with the use of landmarks.

Explanation

The sacral canal results from the fusion of the laminae of the five sacral vertebrae in the midline. Failure of fusion of the S4 and S5 results in the formation of the sacral hiatus. The sacral hiatus is easily identifiable as a small depression between the sacral cornua, hence making the performance of the sacral block possible with the use of landmarks.

The posterior superior iliac spines and sacral hiatus form an equilateral triangle pointing downwards and can be felt in the lateral and prone positions.

The dural sac ends at the level of S2 in adults and S3 in children.

41
Q

Regarding temperature measurement devices and the relevant physical principles on which their function depends, are the following associations true or false?

True / False
The mercury thermometer and thermal expansivity
The metal oxide thermistor and the Wheatstone bridge
The platinum wire and electrical resistance
The thermocouple and the Seebeck effect
The infra red thermometer and the Stefan-Boltzmann equation

A

True
False
True
True
True

Explanation

The correct principle utilised by the metal oxide thermistor is a change in electrical resistance. The thermistor may be used as part of a Wheatstone bridge but this is not the principle of its function.

The liquid expansion thermometers work on the principle of thermal expansitivity, but are limited by their narrow temperature ranges for use.

The resistance of a platinum wire increases linearly over a narrow range, beyond which it is not reliable.

Infra-red thermometry works on the principle of the emitted infrared waves being directly related to the fourth power of the temperature.

42
Q

Which one of the following structures passes through the superior orbital fissure?

Optic nerve
Inferior ophthalmic vein
Lacrimal nerve
Zygomatic nerve
Maxillary nerve

A

Lacrimal nerve

Key learning points
Anatomy
The superior orbital fissure transmits several important structures, including the oculomotor nerve, trochlear nerve, abducens nerve, and branches of the ophthalmic nerve, which are essential for eye movement and sensation.

Explanation

The lacrimal nerve, a branch of the ophthalmic nerve (CN V1), passes through the superior orbital fissure.

The maxillary nerve (CN V2) passes through the foramen rotundum, not the superior orbital fissure.

The inferior ophthalmic vein passes through the inferior orbital fissure, not the superior orbital fissure.

The optic nerve (CN II) passes through the optic canal, not the superior orbital fissure.

The zygomatic nerve, a branch of the maxillary nerve (CN V2), passes through the inferior orbital fissure, not the superior orbital fissure.

43
Q

Are the following statements true or false regarding the management of burns patients?

True / False
Parenteral nutrition markedly attenuates the hypermetabolic response
Bacterial translocation from the intestines occurs 24 hours after a burn injury
In a patient with 60% burns, the metabolic rate is four times higher than the normal rate
High protein diets may improve survival
Nursing the patient in a cool environment reduces the hypercatabolic state

A

False
False
False
True
False

Explanation

After suffering a thermal injury the patient rapidly becomes hypercatabolic, with an increased cardiac output and oxygen consumption.

Severe burn injuries are associated with a greater hypermetabolic response. With a 60% total body surface area burn the metabolic rate is about twice the normal rate (not four times greater).

Patients should be nursed in temperatures of at least 30°C to reduce energy expenditure (not cool environments). The resetting of hypothalamic thermoregulation results in a 1-2°C rise in core temperature. The burned area should be covered to reduce evaporative loss of fluids.

The barrier function of the intestine is lost immediately after a thermal injury, allowing the translocation of bacteria and endotoxins, which can occur within hours (not 24 hours after the burn).

Early enteral feeding preserves intestinal mucosal integrity and prevents translocation of microorganisms into the circulation. Enteral nutrition is associated with a marked attenuation of the hypermetabolic response to a burn injury (not parenteral nutrition).

High protein diets (with a calorie to nitrogen ratio of 100:1), may improve survival after a burn. Fifty per cent of the calories should be in the form of carbohydrate, 30% as lipids or fat and up to 20% as protein or amino acids.

Despite the associated risk of infection supplementary parenteral feeding may be required.

44
Q

A transformer has stepped down a potential difference of 240 V from a central power source to 12 V. It is powering a light bulb in a simple electric circuit. Within the circuit are two resistors in parallel. One resistor is 1 Ω (ohm) and the other is 2 Ω (ohm).

Which one of the following options represents the current flowing through the light bulb?

12A
14A
10A
16A
18A

A

18A

Physics and clinical measurement
Within simple circuit with resistors in parallel. The total resistance can be derived by adding the reciprocals of the individual resistance values.

Explanation

Potential difference (V) = current (I) x resistance (R)

Rearranging the equation:

I = V/R

where:

V = 12
R1 = 1
R2 = 2
With resistor in parallel 1/R = 1/R1 + 1/R2

1/R = 1/1 + 1/2 therefore 1/R = 1 + 0.5

1/R = 1.5

R = 1/1.5 = 0.667

so:

I = 12/0.667 = 18A.

45
Q

Oxygen:

True / False
Has a critical temperature of 183°C
Is stored in cylinders with a white and black shoulder and a French grey body
Percentage in the air is reduced at high altitude
Can be liquefied at room temperature
Forms molecules containing either two or three atoms

A

False
False
False
False
True

Key learning points
Physics and clinical measurement

The critical temperature of a gas is that at which it cannot be liquefied no matter how much pressure is applied. The critical temperature of oxygen is minus 118.6 degreeC.

Explanation

The critical temperature of a gas is that at which it cannot be liquefied no matter how much pressure is applied. The critical temperature of oxygen is −118.6°C. Liquid oxygen cannot be liquefied at room temperature by pressure alone.

Medical oxygen is stored in cylinder with a white shoulder and black body. Medical air is stored in cylinders with a white and black shoulder and a French grey body.

The fractional concentration of oxygen is the same at altitude but its partial pressure falls.

Oxygen is O2 and ozone is O3.

45
Q

Are the following statements true or false concerning red blood cells?

True / False
Are released from the bone marrow as mature erythrocytes
Measure 15 μm in diameter
Have a life span of 120 days in the circulation
Do not contain mitochondria
Express HLA class II antigens on the cell surface

A

False
False
True
True
False

Explanation

Each red blood cell is about 7.5 μm in diameter and 2 μm thick. They are anucleate cells that do not contain mitochondria.

In humans, they survive in the circulation for an average of 120 days. They are initially released from the bone marrow as reticulocytes, which contain residual ribosomal RNA and are still able to synthesise Hb.

The membranes of erythrocytes contain a variety of blood group antigens, but not HLA class II antigens.

46
Q

Patients fitted with permanent cardiac pacemakers:

True / False
Can have an MRI scan provided the pacemaker is MRI-conditional
Require a preoperative 12 lead ECG
Require a preoperative chest x ray
Can have a ventilation/perfusion scan
Can have a CT scan

A

True
True
False
True
True

Explanation

Patients fitted with permanent cardiac pacemakers can safely have a CT scan or a ventilation/perfusion scan.

Cardiac pacemakers and implantable cardiac defibrillators (ICDs) have been suggested to present potential problems to patients undergoing MR procedures from various mechanisms, these include:

Electromagnetic interference
Movement and/or vibration of the pulse generator or lead(s)
Modification of pacemaker function (i.e., damage) of the device
Inappropriate sensing, triggering, or activation of the device
Heating of the leads
Induction of currents in the leads
In order to avoid pacemaker malfunction and injury to the patient, an MRI scan is relatively contraindicated. There is compelling evidence that “modern-day” pacemakers (i.e. devices with decreased ferromagnetic components and more sophisticated circuitry) suggest that certain patients may undergo MRI without harmful effects, provided specific guidelines to minimize or prevent risks are adhered to.

In recent years, a subset of pacmakers have been designed to be safe in an MRI environment. These devices are labelled MRI-conditional. There is increasing evidence to suggest that older “legacy” pacemakers and ICDs implanted after 2008 are safe in an MRI environment provided the device is checked prior to a scan and its function monitored during the procedure.

Important information about the intrinsic cardiac rhythm, the type of pacemaker and how it has been programmed can all be obtained from a 12-lead ECG.

A chest x ray can be used to confirm the location and number of pacemaker wires within the heart, but is not an essential investigation.

47
Q

Which of the following is true of the Mapleson anaesthetic breathing circuits?

True / False
Mapleson E is efficient at gas flow equivalent to the patient’s minute ventilation
The Jackson-Rees modification consists of closed ended reservoir bag
Mapleson A is most efficient for spontaneous ventilation
Mapleson B is efficient for both spontaneous and positive pressure ventilation
The Bain circuit is a modification of the Mapleson A

A

False
False
True
False
False

Key learning points
Physics and clinical measurement

The various breathing systems as classified by Mapleson vary in their efficiency during spontaneous ventilation (SV) and positive pressure ventilation (PPV).

Explanation

The various breathing systems as classified by Mapleson vary in their efficiency during spontaneous ventilation (SV) and positive pressure ventilation (PPV):

Mapleson A - efficient for spontaneous breathing requiring gas flow at the patient’s alveolar volume (70 ml/kg/min)
Mapleson B and C - inefficient for both SV and PPV; requires gas flow of two to three times minute volume (100 ml/kg/min)
Mapleson D - efficient for PPV at gas flow equivalent to patient’s minute volume; the Bain’s circuit is a coaxial version of the Mapleson D
Mapleson E and F - for paediatric use; requires gas flow at two to three times the patient’s minute volume. The Mapleson F consists of an open-ended reservoir bag (Jackson-Rees modification).

48
Q

Are the following statements true concerning aldosterone?

True / False
Is the principal mineralocorticoid secreted by the adrenal medulla
Secretion is increased by a low potassium intake
Angiotensin II is the most important regulator of secretion
In the kidney, has an action on the distal convoluted tubule
Production is reduced in normal pregnancy

A

False
False
True
True
False

Explanation

Aldosterone stimulates the kidney to conserve sodium (Na+) and excrete potassium (K+).

As progesterone increases in pregnancy it stimulates the loss of sodium in the urine. This in turn increases aldosterone production to conserve sodium.

Angiotensin II is the most important regulator of secretion.

It is the principal mineralocorticoid secreted by the adrenal cortex and acts on the distal convoluted tubule and the collecting ducts.

49
Q

Are the following statements concerning thiazide diuretics true?

True / False
Are more potent than loop diuretics.
Reduce renal clearance of urate.
Increase plasma cholesterol levels.
Reduce systemic vascular resistance.
Increase insulin secretion.

A

False
True
True
True
False

Explanation

Thiazide diuretics primarily inhibit sodium transport in the distal tubule, the connecting segment at the end of the distal tubule, and possibly the early cortical collecting tubule (although this finding is controversial).

These segments normally reabsorb less of the filtered load than does the loop of Henle; as a result the thiazide-type diuretics are generally less potent than loop diuretics and, when given in maximum dosage, inhibit the reabsorption of at most 3-5% of the filtered sodium.

As well as the diuretic effect, thiazides have a number of other interesting metabolic effects; a reduction in insulin secretion and reduced glyconeogenesis, coupled with enhanced gluconeogenesis lead to raised plasma glucose levels - most noticeable in diabetic patients.

A modest increase in plasma cholesterol and triglyceride levels is also seen.

Thiazides are notorious for inducing and exacerbating gout.

Diuretics decrease urate excretion by increasing net urate reabsorption; this can occur either by enhanced reabsorption or by reduced secretion. Regardless of the mechanism, volume depletion appears to play an important role in this response, since urate retention does not occur if the diuretic-induced fluid losses are replaced.

The mechanism of reduction in systemic vascular resistance, although widely acknowledged, is still uncertain.

50
Q

Which of the following is normally in a higher concentration outside the cell than in the intracellular fluid?

Adenosine triphosphate (ATP)
Adenosine monophosphate (AMP)
Magnesium
Potassium
Sodium

A

Sodium

Key learning points
Physiology and biochemistry
Sodium is a primarily extracellular ion

Explanation

The following are found in higher concentrations intracellularly than extracellularly:

Potassium
Magnesium
ATP
Adenosine diphosphate (ADP)
AMP, and
Phosphate.

Sodium is a primarily extracellular ion.

51
Q

Is it true or false that galactorrhoea may be caused by the following?

True / False
Haloperidol
Opioids
Cimetidine
Spironolactone
Bromocriptine

A

True
True
True
True
False

Explanation

Haloperiodol, cimetidine may cause galactorrhoea.

Opioids in general have dose dependent effects on hypothalamic function, acutely raising prolactin, growth hormone, thyroid stimulating hormone and adrenocorticotrophic hormone, and inhibiting gonadotrophin releasing hormone acutely and chronically

Bromocriptine and quinagolide are central dopamine receptor agonists that also supress the release of prolactin. They are used in the suppression of lactation.

Spironolactone is known to cause gynaecomastia in men and galactorrrhoea in women

52
Q

A 23-year-old female is having a knee arthroscopy under general anaesthesia.

Anaesthesia is induced with fentanyl 1 mcg/kg and 2 mg/kg propofol. A supraglottic airway is inserted and anaesthesia maintained with an air oxygen mixture and 2.5% sevoflurane. The patient is allowed to breathe spontaneously through a Bain circuit. The fresh gas flow is 9 litres per minute. Over a period of 30 minutes the end-tidal CO2 rises from 4.5 kPa to 8.4 kPa. The capnograph has a baseline reading of 0 kPa.

Which one of the following is the most likely cause of the hypercarbia?

Release of the tourniquet
Hypoventilation
Inadequate fresh-gas flow
Exhausted sodalime
Malignant hyperthermia

A

Hypoventilation

Key learning points
Physics and clinical measurement

During anaesthesia a rise in end-tidal carbon dioxide is common in a spontaneously breathing patient. Having excluded rebreathing and increased equipment dead space, drug induced respiratory depression and hypoventilation is the most likely cause

Explanation

Hypoventilation from the respiratory depressant effects of the opioid and sevoflurane are the commonest and most likely cause of a gradual rise in end-tidal CO2 (EtCO2) occurring during anaesthesia in a spontaneously breathing patient.

If the EtCO2 shows further progressive rise then other clinical signs of malignant hyperthermia should be sought.

Exhausted sodalime and inadequate fresh gas flow into the Bain circuit are causes of rebreathing and a rise in the baseline of the capnograph.

Deflation of the tourniquet would cause a sudden rise in EtCO2.

53
Q

Are the following diuretics correctly linked with their main site of action?

True / False
Mannitol and the descending limb of the loop of Henle
Spironolactone and the distal convoluted tubule and collecting duct
Furosemide and thick ascending limb of the loop of Henle
Metolazone and the early distal tubule
Bendroflumethiazide and the proximal tubule

A

False
True
True
True
False

Explanation

Thiazides, of which metolazone and bendroflumethiazide are examples, inhibit sodium reabsorption at the distal convoluted tubule.

Loop diuretics inhibit reabsorption from the ascending limb of the loop of Henle.

Spironolactone is a synthetic steriod with an aldosterone-like structure which antagonises the effect of aldosterone on the distal convoluted tubule and collecting duct. It also has anti-androgen effects.

Mannitol is an osmotic diuretic and works primarily on the proximal convoluted tubule but also has secondary effects on the descending loop of Henle and collecting ducts.

54
Q

For a patient with a known history of coronary artery disease undergoing major vascular surgery, which GDHT parameter is most crucial to monitor intraoperatively to reduce the risk of myocardial ischemia?

Central venous oxygen saturation (ScvO2)
Pulmonary artery occlusion pressure (PAOP)
Cardiac index (CI)
Left ventricular end-diastolic volume (LVEDV)
Arterial blood lactate levels

A

Cardiac index

Key learning points
Physics and clinical measurement

Monitoring and optimising cardiac index (CI) during goal-directed haemodynamic therapy (GDHT) in patients with known coronary artery disease undergoing major surgery is crucial to ensure adequate myocardial oxygen delivery and reduce the risk of intraoperative myocardial ischemia.

Explanation

Monitoring and optimising CI can help ensure adequate myocardial oxygen delivery by maintaining sufficient cardiac output, which is crucial in patients with coronary artery disease undergoing major surgery to reduce the risk of myocardial ischemia.

ScvO2 is a marker of global oxygen balance but may not directly reflect myocardial oxygen demand and supply, especially in patients with localised coronary artery disease.

PAOP provides an estimate of left ventricular filling pressure but may not accurately predict fluid responsiveness or myocardial oxygen demand in all patients, particularly those with ventricular dysfunction or valvular heart disease.

LVEDV is a determinant of preload but measuring it directly is challenging in the intraoperative setting, and it does not provide a complete picture of myocardial oxygen demand and supply.

Elevated lactate levels can indicate systemic hypoperfusion but are a late marker of tissue hypoxia and may not specifically guide the management of myocardial ischemia risk.

55
Q

A 74-year-old man is admitted to the Intensive care unit following an emergency laparotomy for faecal peritonitis.

He is intubated, sedated and ventilated. His vital signs are: temperature 37.8°C, pulse rate 110 beats per minute, BP 130/90 mmHg, Central venous pressure 15 mmHg and urine output 20 ml/hr.

Which one of the following drug infusion regimens is most likely to improve hepatosplanchnic and renal blood flow?

Dopamine 15 mcg/kg/minute
Dopexamine 1 mcg/kg/minute
Isoprenaline 2 mcg/kg/minute
Adrenaline 0.05 mcg/kg/minute
Noradrenaline 0.5 mcg/kg/minute

A

Dopexamine 1 mcg/kg/minute

Key learning points
Pharmacology

Inotropes that have dopaminergic (DA1 agonist properties) improve splanchnic and renal blood flow. They do not prevent renal failure in high risk patients.

Explanation

Dopexamine hydrochloride is a synthetic catecholamine and is a structural analogue of dopamine. It has marked intrinsic activity at the beta-2 adrenoreceptor and to a lesser extent dopaminergic - 1 (DA1) and dopaminergic- 2 receptors. It has no alpha-1 adrenoreceptor activity. DA1 and DA2 receptors are present in the smooth muscle of the mesenteric and renal vasculature. Stimulation of these receptors increases vasodilatation and improved blood flow.

Both dopamine and dopexamine have been used to improve urine output (increasing renal blood flow, glomerular filtration and causing a natriuresis) and splanchnic blood flow.

However, low dose infusions of these drugs have not shown to reduce the incidence of renal failure and mortality in high risk patients. Dopexamine will also increase cardiac output and blood flow via beta-1 agonist activity.

Dopamine at an infusion rate of 15 mcg/kg/minute will exert beta-1 and alpha-1 effects and negligible DA1 activity.

Adrenaline and noradrenaline both have beta-1, beta-2 and alpha-1 activity but no dopaminergic activity.

Isoprenaline has beta activity only.

The use of vasopressors may have a detrimental effect on hepatosplanchnic and renal blood flow (vasoconstriction) and at best in fluid resuscitated no significant effects. The addition of a dopamine agonist will improve flow.

56
Q

An 18-year-old man is admitted to the Emergency Department with an acute exacerbation of his asthma. After initial improvement with nebulised salbutamol, he deteriorates.

On examination, heart rate 110 beats per minute, respiratory rate 30 breaths per minute, blood pressure 90/50 mm temperature 37.8°C. Breath sounds are quiet on auscultation and his trachea is central. He is receiving high flow oxygen via a reservoir oxygen mask. Arterial blood gases show pH=7.37, PaCO2 3.0 kPa, Pa02 10 kPa, BE=-5.

Which one of the following is the single most urgent investigation at this point?
Spirometry
Blood cultures
Chest X-ray
Obtain sputum culture
Peak flow rate

A

Chest XR

Key learning points
Other
In the context of a severe asthma exacerbation with acute deterioration, ruling out life-threatening complications such as pneumothorax or pneumonia is paramount. A chest X-ray is the most urgent investigation to provide immediate and crucial diagnostic information.
Explanation

A chest X-ray is crucial in the acute setting to rule out life-threatening complications such as pneumothorax, which can present with a sudden deterioration in a patient with asthma. It helps identify other possible complications like atelectasis or pneumonia that might be contributing to the clinical deterioration. Given the clinical scenario and the patient’s severe symptoms, a CXR is the most urgent investigation to assess these potential issues.

Blood cultures are used to identify any bacterial infection in the bloodstream, which can help guide antibiotic therapy. In the context of an acute asthma exacerbation, an underlying infection can complicate the clinical picture. However, blood cultures are not the most urgent investigation in this case, as the immediate concern is to assess the severity of the asthma exacerbation and any possible complications such as pneumothorax.

Obtaining a sputum culture can help identify any bacterial infection in the respiratory tract. This can be important for guiding antibiotic therapy. However, in the acute setting of an asthma exacerbation with severe respiratory distress, it is not the most urgent investigation compared to imaging that can reveal life-threatening complications.

Peak flow rate measurement can help assess the severity of airway obstruction in asthma. It is a useful tool for monitoring asthma control over time. However, during a severe exacerbation with significant respiratory distress and potential complications, it is not the most urgent investigation. Imaging to exclude life-threatening conditions takes precedence.

Spirometry is a comprehensive test of lung function and is valuable in the long-term management of asthma. However, it is not suitable for acute settings, especially when the patient is in severe respiratory distress. The test requires patient cooperation and can exacerbate symptoms during an acute attack.

57
Q

What is the most appropriate method for decontaminating video laryngoscopes after use to ensure they are safe for subsequent patients?

High-level disinfection using peracetic acid
Simple soap and water cleaning
Wiping down with low-concentration alcohol wipes
Autoclaving at standard sterilisation temperatures
Irradiation with gamma rays

A

High-level disinfection using peracetic acid

Key learning points
Physics and clinical measurement
For video laryngoscopes, high-level disinfection that does not damage the device is critical. Peracetic acid is an excellent choice as it effectively destroys a broad range of pathogens and is less likely to damage sensitive equipment compared to methods like autoclaving or alcohol-based disinfectants.

Explanation

High-level disinfection using peracetic acid:
High-level disinfection using peracetic acid is recommended for videolaryngoscopes. This method is effective at killing a wide range of pathogens, including bacteria, viruses, and fungi, without causing damage to the delicate electronic components and optics of the videolaryngoscopes​​.

Autoclaving at standard sterilisation temperatures:
Autoclaving, while effective for many medical tools, is not suitable for video laryngoscopes as it can damage the sensitive electronic components and optics that are integral to these devices. High temperatures and steam can degrade the materials and impair functionality.

Irradiation with gamma rays:
Gamma irradiation is a sterilisation method used primarily for disposable medical supplies and certain types of equipment that are not sensitive to radiation. It is not typically used for electronic devices such as video laryngoscopes due to potential damage to electronic components.

Simple soap and water cleaning:
While initial cleaning with soap and water is important to remove visible contamination, it does not provide the level of disinfection required for medical devices that come into contact with mucous membranes. This method alone is insufficient for preventing cross-contamination between patients.

Wiping down with low-concentration alcohol wipes:
Low-concentration alcohol wipes might remove some surface bacteria but are not effective against all types of microorganisms, especially viruses and fungi. Moreover, alcohol wipes do not provide a high enough level of disinfection for equipment used in such invasive procedures as videolaryngoscopy.

58
Q

The following arterial blood gases were taken one day after a bowel resection:

pH 7.21 (7.36-7.44)
pCO2 4.2 kPa (4.7-6.0)
pO2 9.6 kPa (11.3-12.6)
Standard bicarbonate 17.6 mmol/L (20-28)
Base excess −6.2 mmol/L (−2 to +2)
Having regard to these results, are the following statements true?

True / False
Immediate management should consist of administration of IV bicarbonate
Hypovolaemia may be a cause
The pCO2 is low due to respiratory compensation
Sepsis may be an underlying cause
There is a metabolic alkalosis

A

False
True
True
True
False

Explanation

There is a metabolic acidosis (pH low, base excess low, HCO3 low).

Respiratory compensation for metabolic acidosis consists of hyperventilation eliminating CO2.

As with any critically ill patient immediate management should be to give oxygen and follow the ABC (Airway, Breathing, Circulation) format.

Intravenous bicarbonate is inappropriate.

Sepsis may cause this blood gas picture and should be considered.

Hypovolaemia may cause metabolic acidosis due to inadequate tissue perfusion and subsequent anaerobic metabolism and should also be considered.

59
Q

An intravenous induction agent is presented to you. It has the following characteristics:

A cyclohexanone ring with one chiral centre and is presented as a racemic mixture
Local anaesthetic properties.
Which of the following best describes its primary mechanism of action?

Irreversible competitive antagonist affecting Ca2+ channels
Agonist affecting Cl- channels
Non-competitive antagonist affecting Ca2+ channels
Inverse agonist affecting Cl- channels
Reversible competitive antagonist affecting Na+ channels

A

Non-competitive antagonist affecting Ca2+ channels

Key learning points
Pharmacology
Ketamine is a cyclohexanone derivative that is a non-competitive antagonist affecting Ca2+ channels.

Explanation

The drug in question is ketamine. As an intravenous induction agent its structure and pharmacodynamic effects are unique. The molecule is based on a cyclohexanone ring (2-(O-chlorophenyl)-2-methylamino cylohexanone). Ketamine acts mainly on the brain and spinal cord but also has local anaesthetic properties.

It is primarily a non-competitive antagonist for the N-D-methyl-aspartate (NMDA) receptor affecting Ca2+ .channels. It also has local anaesthetic activity by affecting neuronal Na+ channels.

Ketamine produces a profound state of dissociative anaesthesia (profound amnesia and analgesia) and sedation.

An example of an irreversible competitive antagonist includes phenoxybenzamine, an alpha-1 adrenoreceptor antagonist. It forms a covalent bond with the receptor affecting calcium influx.

Benzodiazepines are agonists affecting chloride influx at the GABAA receptor.

Beta-carboline is an inverse agonist affecting chloride influx at the GABAA receptor.

Ketamine is a cyclohexanone derivative that is a non-competitive antagonist affecting Ca2+ channels.

60
Q

Which one of the following cranial nerves is primarily responsible for the afferent limb of the pupillary reflex to light?

Abducens nerve
Oculomotor nerve
Optic nerve
Trochlear nerve
Trigeminal nerve

A

Optic nerve

Key learning points
Anatomy
The pupillary reflex to light involves the optic nerve (CN II) as the afferent limb, detecting light and transmitting this information to the brain, and the oculomotor nerve (CN III) as the efferent limb, causing pupillary constriction.

Explanation

Optic nerve: The optic nerve (CN II) is responsible for the afferent limb of the pupillary reflex, detecting light and transmitting the signal to the brain.

Abducens nerve: The abducens nerve (CN VI) controls the lateral rectus muscle and is not involved in the pupillary reflex to light.

Oculomotor nerve: The oculomotor nerve (CN III) controls the efferent limb of the pupillary reflex, causing pupillary constriction.

Trochlear nerve: The trochlear nerve (CN IV) controls the superior oblique muscle and is not involved in the pupillary reflex to light.

Trigeminal nerve: The trigeminal nerve (CN V) provides sensory innervation to the face and is not involved in the pupillary reflex to light.

61
Q

Many processes we deal with in anaesthesia involve a relationship between one variable and another.

Which one of the following relationships is linear?

The relationship between the junction potential and temperature in a copper/constantan thermocouple
The relationship between the pressure of a fixed mass of gas and its volume at a fixed temperature
The relationship between efficacy and log-dose of a pure agonist on mu receptors
The relationship between a decrease in plasma concentration of an intravenous bolus of a drug with time for a single compartment model
The relationship between current and degrees from a mains electricity source

A

The relationship between the junction potential and temperature in a copper/constantan thermocouple

Key learning points
Physics and clinical measurement
A thermojunction voltage of a thermocouple is generated at the tip that is directly proportional to temperature. This should not be confused with a change in resistance with temperature.

Explanation

A thermocouple is composed of two bonded wires of dissimilar metals, iron/constantan or copper/constantan (constantan is an alloy of copper and nickel). A thermojunction voltage is generated at the tip that is directly proportional to temperature (Seebeck effect).

All the other relationships are not linear.

The relationship between a decrease in plasma concentration of an intravenous bolus of a drug with time for a single compartment model is a washout exponential.

The relationship between current and degrees or time from a mains electricity source is a sine wave.

The relationship between efficacy and log-dose of a pure agonist on mu receptors is a sigmoid curve.

The relationship between the pressure of a fixed mass of gas and its volume (Boyle’s law) at a fixed temperature is an inverse relationship and produces a hyperbolic curve.

62
Q

Are the following statements true or false of metoclopramide?

True / False
May cause drowsiness.
Increases barrier pressure.
Increases pyloric tone.
Is an antagonist at the 5-hydroxytryptamine (5-HT) receptor.
Causes side effects which are treated with antimuscarinic drugs.

A

True
True
False
True
True

Explanation

Although it primarily has an antagonistic action at dopamine receptors, metoclopramide is a 5-hydroxytryptamine (5-HT) antagonist at higher doses. Its extrapyramidal side effects can be treated with procyclidine which has antimuscarinic properties.

Metoclopramide is relatively free of side effects, apart from oculogyric crises (an extrapyramidal effect) and drowsiness.

The peripheral effects of metoclopramide inhibiting the action of dopamine are illustrated by its enhancement of antral and fundal contractility and relaxation of the pylorus. The net effect is to increase the rate of gastric emptying.

The lower oesophageal sphincter pressure is, however, increased and this leads to an increase in barrier pressure.

63
Q

Which pathway best describes the normal flow of CSF from its point of production to its exit into the systemic circulation?

Third ventricle -> lateral ventricles -> fourth ventricle -> choroid plexus -> subarachnoid space -> arachnoid villi
Choroid plexus -> fourth ventricle -> third ventricle -> lateral ventricles -> subarachnoid space -> dural sinuses
Lateral ventricles -> third ventricle -> choroid plexus -> fourth ventricle -> dural sinuses -> subarachnoid space
Choroid plexus -> lateral ventricles -> third ventricle -> fourth ventricle -> subarachnoid space -> arachnoid villi
Subarachnoid space -> lateral ventricles -> third ventricle -> fourth ventricle -> choroid plexus -> arachnoid villi

A

Choroid plexus -> lateral ventricles -> third ventricle -> fourth ventricle -> subarachnoid space -> arachnoid villi

Key learning points
Anatomy

Understanding the correct pathway of CSF flow is crucial for managing conditions like hydrocephalus and for performing procedures like lumbar puncture or placing shunts. Accurate knowledge helps in diagnosing pathologies correctly and planning effective intervention strategies.

Explanation

Choroid plexus -> lateral ventricles -> third ventricle -> fourth ventricle -> subarachnoid space -> arachnoid villi:
This option correctly describes the typical pathway of CSF flow. CSF is produced mainly by the choroid plexus located in the lateral and fourth ventricles. It flows through the lateral ventricles to the third and then to the fourth ventricle, passing through the cerebral aqueduct. It then moves into the subarachnoid space surrounding the brain and spinal cord, and finally, CSF is absorbed into the venous system via the arachnoid villi into the dural sinuses.

Choroid plexus -> fourth ventricle -> third ventricle -> lateral ventricles -> subarachnoid space -> dural sinuses:
This option incorrectly suggests that CSF flows from the fourth ventricle to the third and then to the lateral ventricles, which is anatomically incorrect as it reverses the natural downstream flow.

Lateral ventricles -> third ventricle -> choroid plexus -> fourth ventricle -> dural sinuses -> subarachnoid space:
This sequence is incorrect as it implies that the choroid plexus is a point through which CSF flows after leaving the third ventricle, which is not correct. It also incorrectly lists the dural sinuses before the subarachnoid space.

Subarachnoid space -> lateral ventricles -> third ventricle -> fourth ventricle -> choroid plexus -> arachnoid villi:
This option incorrectly reverses the entire pathway of CSF circulation, beginning with the subarachnoid space and moving backwards through the ventricular system.

Third ventricle -> lateral ventricles -> fourth ventricle -> choroid plexus -> subarachnoid space -> arachnoid villi:
This option also inaccurately portrays the flow of CSF, suggesting an upstream movement from the third to the lateral ventricles and misplacing the choroid plexus in the pathway.

64
Q

In immunology, are the following statements true?

True / False
Hereditary angio-oedema is due to deficiency of C1 esterase inhibitor
Degranulation of mast cells is caused by crosslinking of IgM by antigen
Mast cells are found in the respiratory mucosa
All of the IgG subclasses can cross the placenta
Leukotrienes are products of the cyclo-oxygenase pathway

A

True
False
True
True
False

Explanation

All four sub-classes of IgG are able to cross the normal placenta.

Leukotrienes are products of the 5-lipoxygenase pathway.

Prostaglandins are the products of the cyclo-oxygenase pathway, which is the other arm of arachidonic acid metabolism.

Degranulation of mast cells is caused by crosslinking of IgE (not IgM).

Mast cells are the tissue equivalent of basophils and are found in the lungs, skin and gastrointestinal tract. They are the principal mediators of immediate hypersensitivity reactions.

Hereditary angio-oedema is an autosomal dominant condition associated with a quantitative or qualitative deficiency of C1 esterase inhibitor.

65
Q

In which one of the following situations is a calibrated LiDCO system most accurate?

An adult patient in cardiogenic shock on an aortic balloon pump
An adult patient with aortic regurgitation
Immediately after a bolus of vecuronium
A patient in the second trimester of pregnancy
An adult patient with manic depression on oral lithium therapy

A

A patient in the second trimester of pregnancy

Key learning points
Physics and clinical measurement

Contraindications and sources of inaccuracy in LiDCO cardiac output monitoring include the first trimester of pregnancy, weight <40kg, aortic regurgitation, intra-aortic balloon pump use, and therapeutic lithium administration.

Explanation

LiDCO is a method of cardiac output monitoring in which pulse contour analysis is used to infer haemodynamic variables, after calibration using 0.15-0.3 mmol lithium chloride injected via a central line.

The dose of lithium marker is very small and has no known pharmacological effects. However it is contraindicated during the first trimester of pregnancy and in patients weighing less than 40 kg. Concurrent oral lithium treatment prevents accurate calibration. The results are not considered valid in patients with aortic regurgitation or those undergoing intra-aortic balloon pump therapy. Depolarising and non-depolarising muscle relaxants have a positively charged ion (quaternary ammonia ion) that can be detected by the LiDCO lithium sensor so 15-30 minutes should elapse before calibrating.

66
Q

Do the following have an antiemetic action?

True / False
Hyoscine hydrobromide
Morphine sulphate
Perphenazine
Chlorpropamide
Promethazine hydrochloride

A

True
False
True
True
True

Explanation

Hyoscine is an alkaloid.

Promethazine is an antihistamine.

Perphenazine is an antipsychotic.

All three have antiemetic properties.

The sulphonylurea chlorpropramide may have antiemetic properties and has thus been marked as true.

Morphine has an emetic action.

67
Q

Are the following fundamental (base) SI units?

True / False
A metre is the unit of length
A gram is the unit of mass
A newton is the unit of power
A watt is the unit of energy
A hertz is the unit of frequency

A

True
False
False
False
False

Explanation

The system of units or SI (Système International d’unités) was introduced in 1960 and is based on the metric system.

There are seven base units:

Metre
Second
Kilogram
Ampere
Kelvin
Candela
Mole.

Derived units include the newton, pascal, joule, watt and hertz.

68
Q

What is a key advantage of using ultrasound guidance in regional anaesthesia?

Applicable for all types of patients
Decreased procedural time
Elimination of procedural risks
Improved visualisation of target structures
Reduced need for patient monitoring

A

Improved visualisation of target structures

Key learning points
Other
The primary advantage of using ultrasound guidance in regional anaesthesia is the improved visualisation of anatomical structures, which enhances the accuracy and safety of the procedure. It’s important to understand that while this technique offers significant benefits, it does not eliminate the need for patient monitoring or all procedural risks, nor does it universally decrease procedural time.

Explanation

Improved visualisation of target structures is the correct answer. Ultrasound allows for real-time visualisation of the needle, target nerves, surrounding tissues, and vessels, which can improve the accuracy of needle placement. This can lead to a higher success rate of nerve blocks, better quality of the block, and potentially reduced incidence of complications such as nerve damage or intravascular injection.

While ultrasound guidance can be beneficial for a wide range of patients, there are some limitations and contraindications. For example, in patients with certain skin infections or anatomical abnormalities, ultrasound-guided procedures might be more challenging or not recommended.

While ultrasound-guided techniques can enhance the safety and effectiveness of regional anaesthesia, they do not reduce the need for patient monitoring. Continuous monitoring before, during, and after the procedure is essential to ensure patient safety and manage any complications that may arise.

Although ultrasound guidance can improve the accuracy of needle placement, it may not necessarily decrease the procedural time. In fact, in some cases, the procedure may take longer, especially if the practitioner is less experienced with ultrasound techniques. Over time and with increased proficiency, the procedural time may decrease.

While ultrasound guidance can reduce the risk of certain complications associated with regional anaesthesia, it does not eliminate all procedural risks. Complications, though less likely, can still occur, such as infection, bleeding, or allergic reactions to the anaesthetics used.

69
Q

Are the following true or false of glycolysis?

True / False
Has a net gain of 9 molecules of ATP for every molecule of glucose consumed
Occurs within the cytoplasm of the cell
Acetyl co-enzyme A is the main cofactor
Involves the conversion of lactate to pyruvate
Does not consume oxygen

A

False
True
False
False
True

Key learning points
Physiology and biochemistry
Anaerobic metabolism is much less efficient than aerobic metabolism and results in the production of lactate.

Explanation

Acetyl co-enzyme A is involved in the citric acid cycle rather than glycolysis.

Glycolysis is the process in which glucose is broken down to pyruvate. For each molecule of glucose it consumes 2 molecules of ATP and generates 4. Therefore, there is a net gain of 2 ATP molecules per molecule of glucose.

During anaerobic metabolism pyruvate is reduced to lactate (not lactate to pyruvate) to regenerate NAD+ and allow glycolysis to continue.

Glycolysis takes place in the cytoplasm and does not consume oxygen or produce carbon dioxide.

70
Q

Which of one of the following statements about the cricoid cartilage is correct?

A force of 4 newtons must be applied to the cricoid cartilage to control passive regurgitation of gastric contents
Represents the narrowest part of the airway in adults
Regurgitation of gastric contents is prevented by the Sellick manoeuvre
The upper border is attached to the hyoid bone
The lower border is attached to the first tracheal ring

A

Represents the narrowest part of the airway in adults

Key learning points
Anatomy
The cricoid cartilage is attached to the thyroid cartilage at the upper border and the first tracheal ring at the lower border, and is the narrowest part of the airway in children. Cricoid pressure of 40 N is recommended to reduce the risk of aspiration during anaesthesia.

Explanation

The upper border of the cricoid cartilage is attached to the thyroid cartilage and the lower border is attached to the first tracheal ring. In adults the narrowest part of the airway is at the glottis, and in children it is at the level of the cricoid cartilage. The upper border of the cricoid cartilage is attached to the thyroid cartilage (not hyoid bone) and the lower border is attached to the first tracheal ring. The application of cricoid pressure to control regurgitation (or reduce the risk of aspiration) of gastric contents during the induction of anaesthesia, was described by Sellick in 1961.There have been reports of pulmonary aspiration despite what was thought to be effective cricoid pressure. One possible reason is that the oesophagus is not exactly posterior to the cricoid and the manoeuvre might not necessarily result in midline esophageal compression. Based on studies of cricoid force to prevent material from reaching the pharynx, 40 N (10 N = 1.0 kg) was recommended.

71
Q

This set of arterial blood gases has been taken from a 75-year-old male with an acute exacerbation of chronic obstructive airway disease (COPD). He is breathing air.

His respiratory rate is 30 breaths per minute, pulse rate 110 beats per minute and blood pressure 160/80 mmHg.

pH 7.23 (7.36-7.44)
PaO2 77 mmHg (10.8 kPa) (11.3-12.6 kPa)
PaCO2 62 mmHg (8.2 kPa) (4.7-6.0 kPa)
BE +3 mmol/L (± 2)
Standard bicarbonate 29 mmol/L (20-28)
SaO2 91%

What is the single most likely direct central influence on his respiratory rate?

Arterial pH
Arterial PaO2
CSF pH
Arterial PaCO2
CSF PCO2

A

CSF pH

Key learning points
Physiology and biochemistry
The central chemoreceptors respond to changes in CSF pH and not CO2

Explanation

The blood gas picture is one of an acute respiratory acidosis.

The central chemoreceptors are located in the ventral surface of the medulla and are particularly sensitive to changes in blood PCO2 tension. They respond primarily to changes in CSF pH and indirectly to changes in PaCO2.

CO2 readily diffuses across the blood brain barrier and combines with water to produce carbonic acid (H2CO3). This reaction is catalysed by the ezyme carbonic anhydrase. The carbonic acid subsequently dissociates to to H+ and HCO3- ions. The CSF has less buffering capacity than plasma and a fall in CSF pH will then cause reflex respiratory stimulation.

The central chemoreceptors are not stimulated by a fall in PaO2.

The peripheral chemoreceptors are the carotid bodies located at the bifurcation of the common carotid artery and the aortic bodies lie in the arch of aorta. The carotid bodies are sensitive to changes in arterial PaCO2, PaO2 and pH whereas the aortic bodies are principally sensitive to changes in arterial PaO2 and PaCO2 only.

72
Q

Are the following statements true of thiazide diuretics?

True / False
Are typically given intravenously
Typically cause hypocalcaemia
Are associated with hypomagnesaemia
Reduce extracellular fluid volume
Are associated with a metabolic alkalosis

A

False
False
True
True
True

Explanation

Thiazide diuretics act by inhibiting the luminal transmembrane-coupled NaCl transport system in the early segment of the distal convoluted tubule.

The inhibition of sodium reabsorption and stimulation of chloride excretion, together with excretion of sodium and potassium, and reduced excretion of bicarbonate, results in an increase in water excretion.

The reduction in extracellular fluid volume (blood volume and cardiac output) leads to the activation of the renin-angiotensin-aldosterone and adrenergic systems in an attempt to re-establish central blood volume. Eventually blood volume and cardiac output return to normal but the peripheral resistance is reduced.

The activation of these counter-regulatory systems may take up to nine days and explains their variable interpersonal antihypertensive efficacy.

Thiazides have many side effects, some of which include:

Hypokalaemia
Impaired glucose tolerance
Hypercalcaemia
Hypomagnesaemia
Hypochloraemic alkalosis (due to renal loss of acid)
Gout
Photosensitivity
Rashes
Postural hypotension
Aplastic anaemia.

Thiazide diuretics are oral preparations.

73
Q

Are the following derived values?

True / False
Systemic vascular resistance (SVR)
Ejection fraction
Pulmonary capillary wedge pressure (PCWP)
Mixed venous oxygen saturation
Cardiac index

A

True
True
False
False
True

Explanation

Directly measured values are:

Pulmonary capillary wedge pressure (PCWP)
Mean pulmonary artery pressure (PAP)
Mixed venous oxygen saturation (SvO2)
Central venous pressure
Heart rate
Stroke volume.
Derived values include:

Systemic vascular resistance (SVR)
Stroke volume

and anything that is indexed.

Please note that stroke volume can be measured directly or indirectly depending on the monitor used:

Echocardiography - direct (some might argue this is indirect)
Changes in bioimpedence - indirect

74
Q

A 63-year-old woman presents to the chronic pain clinic with trigeminal neuralgia affecting the mandibular division of the nerve.

Through which one of the following structures does this nerve exit the skull?

Foramen lacerum
Stylomastoid foramen
Superior orbital fissure
Foramen ovale
Foramen rotundum

A

Foramen ovale

Key learning points
Anatomy
The mandibular division of the trigeminal nerve leaves the middle cranial fossa through the foramen ovale with the lesser petrosal nerve.
Explanation

The trigeminal nerve is a mixed nerve providing sensory supply to the face and motor supply to the muscles of mastication, anterior digastric, mylohyoid, tensor tympani and tensor palati.

It has ophthalmic (V1), maxillary (V2) and mandibular (V3) divisions.

The ophthalmic division divides into lacrimal, frontal and nasociliary branches before exiting the skull via the superior orbital fissure.

The maxillary division emerges from the skull through the foramen rotundum, while the mandibular division transits the foramen ovale.

The stylomastoid foramen conveys the facial nerve and the foramen lacerum small meningeal branches of the ascending pharyngeal artery and emissary veins.

75
Q

In laser surgery, what is the primary advantage of using a laser with a wavelength that is highly absorbed by water?

It allows for precise cutting and ablation of tissue with minimal thermal damage to surrounding areas
It enhances the depth of laser penetration for treatment of deeper tissues
It promotes faster healing by minimising direct tissue contact
It increases the speed of surgical procedures by maximising energy absorption
It reduces the risk of laser light being scattered by tissue, improving precision

A

It allows for precise cutting and ablation of tissue with minimal thermal damage to surrounding areas

Key learning points
Physics and clinical measurement
Lasers with wavelengths highly absorbed by water are ideal for precise surgical applications, as they allow for rapid vaporisation of tissue with minimal thermal spread, thus reducing collateral damage.

Explanation

Lasers that are highly absorbed by water are excellent for precise cutting and ablation because they are absorbed by the water in the cells, causing rapid heating and vaporisation of the cell, with minimal heat spread to adjacent tissues. The CO2 laser is absorbed by water.

High absorption by water actually limits the depth of penetration because the laser energy is quickly absorbed near the surface, not allowing it to reach deeper tissues.

While reduced scattering can improve precision, the primary advantage of water absorption is the precise cutting and ablation capability, not reduced scattering.

While efficiency is important, the key advantage is the precision and control over thermal damage, not necessarily the speed of the procedure.

Minimising direct tissue contact is more related to the non-contact nature of laser surgery in general, rather than the specific absorption by water.

76
Q

For each of the following statements, select whether they are true or false:

True / False
A line drawn at a point on a curve y=(f)x where dy/dx=0 is a horizontal tangent
If the equation y=3x2 + 4x is differentiated the product is 6x+4
The calculation of cardiac output from a thermodilution curve is an application of differentiation
A picofarad is 10-19 Farad
For the exponential function C=C0.e-kt, the time constant(tau)=0.693/k

A

True
True
False
False
False

Explanation

The relationship between the dependent variable (y) against an independent variable (x) is explored in the Primary FRCA examination:

y is a function (f) of x; y=(f)x

The equation for a straight line is:

y=mx or y=mx+c

The slope or gradient of a straight line (m) can easily be calculated by dividing change in y by change in x. The gradient will be the same at any point of a straight line.

The gradient of a curve is more difficult to determine, as the gradient will differ at each point of a curve. The gradient at any point on that curve will be the rate of change of x divided by the rate of change of x (dy/dx). Any point at which the tangent to the graph is horizontal is called a stationary point. At these stationary dy/dx = 0.

The gradient of a curve can be found by differentiating the formula of the curve. What is the gradient of the curve y = 2x3 at the point (3,54)? There are rules of differentiation. In order to differentiate power the following formula is valid:

If y = kxn, dy/dx = nkxn-1(where k is a constant)

dy/dx = 6 multiplied by x2

When x = 3, dy/dx = 6× 9 = 54 (the gradient)

Differentiating the equation in stem 2

dy/dx=3x2 + 4x = 6x+4 we can now calculate the gradient if we know x.

Calculating the cardiac output from a thermodilution curve uses the calculation of area under a curve between two limits. This process involves integration.

Tau (τ) is the time it would have taken for a negative exponential process were the initial rate of change to be maintained throughout to complete.

τ = 1/k or t1/2 = 0.69τ

t1/2 is the half life and 0.69 is a constant (ln2).

77
Q

Are the following true regarding field block for inguinal hernia repair?

True / False
Prilocaine 0.5% with adrenaline is a suitable choice of local anaesthetic agent
Involves blocking dermatomes S2-S4
Requires blockade of the genital branch of the genitofemoral nerve
Requires blockade of the subcostal nerve
Requires blockade of the iliohypogastric and ilioinguinal nerves

A

True
False
True
True
True

Explanation

A field block for an inguinal hernia repair is ideal for high risk patients unsuited for general or spinal anaesthesia. The innervation of the inguinal region is through the ventral rami of T11 and T12 and the two upper branches of the lumbar plexus, the iliohypogastric and ilioinguinal nerves.

The anterior cutaneous branch of the iliohypogastric nerve supplies the skin above the pubis and medial end of the inguinal ligament. The ilioinguinal nerve supplies the skin over the root of the penis and scrotum.

The ventral ramus of the 12th thoracic or subcostal nerve sends a branch to join the first lumbar root and supplies the skin over the lower anterior abdominal wall. The genital branch of the genitofemoral nerve may supply skin in the medial part of the groin.

Prilocaine 0.5% with adrenaline is a suitable choice of agent, which allows a large volume of solution to be used safely.

78
Q

Are the following statements true regarding 1,25(OH)2 vitamin D?

True / False
Stimulated by hypophosphataemia
25 hydroxylation occurs in the kidney
Promotes phosphate and calcium absorption from the gut
Increased in chronic renal failure
Attaches to intracellular receptors in the nucleus

A

True
False
True
False
True

Explanation

25 hydroxylation occurs in the liver (not kidney). 1 hydroxylation is in the kidney.

Hypophosphataemia and hypocalcaemia are the main stimulants of 1 alpha hydroxylation.

1,25(OH)2 vitamin D promotes phosphate and calcium absorption from the gut, it is a steroid hormone and is an agonist for the calcitriol or vitamin D receptor (VDR) and also known as NR1I1 (nuclear receptor subfamily 1, group I, member 1), is a member of the nuclear receptor family of transcription factors.

It is decreased in chronic renal failure (not increased).

79
Q

Which one of the following is a primary perioperative concern for patients taking chlorpromazine?

Caution with desflurane use
Reduction in anaesthetic agent requirements
Avoidance of indirect-acting sympathomimetics
Prolongation of neuromuscular blockade drugs
Interference with platelet function

A

Caution with desflurane

Key learning points
Pharmacology

Patients taking chlorpromazine require special consideration with the use of desflurane during surgery due to the potential for sensitising the myocardium, which can lead to cardiac complications.

Explanation

Chlorpromazine can sensitise the myocardium to the effects of desflurane, potentially leading to cardiac complications.

Avoidance of indirect-acting sympathomimetics is a concern for patients taking MAOIs, such as phenelzine or moclobemide, due to the risk of hypertensive crisis. It is not relevant for patients taking chlorpromazine.

Interference with platelet function is a concern for patients taking valproate, which can affect platelet aggregation. It is not a concern for patients taking chlorpromazine.

Prolongation of neuromuscular blockade drugs is a concern for patients taking lithium. Chlorpromazine does not significantly affect the duration of neuromuscular blockade.

Reduction in anaesthetic agent requirements is relevant for patients on lithium, as it can reduce the requirement for anaesthetic agents. It is not the primary concern for patients taking chlorpromazine.